Падение напряжения при параллельном соединении: Падение напряжения при параллельном соединении резисторов. Параллельное соединение проводников

Содержание

Падение напряжения при параллельном соединении резисторов. Параллельное соединение проводников

Обычно все затрудняются ответить. А вот загадка эта в применении к электричеству решается вполне определенно.

Электричество начинается с закона Ома.

А уж если рассматривать дилемму в контексте параллельного или последовательного соединений — считая одно соединение курицей, а другое — яйцом, то сомнений вообще нет никаких.

Потому что закон Ома — это и есть самая первоначальная электрическая цепь. И она может быть только последовательной.

Да, придумали гальванический элемент и не знали, что с ним делать, поэтому сразу придумали еще лампочку. И вот что из этого получилось. Здесь напряжение в 1,5 В немедленно потекло в качестве тока, чтобы неукоснительно выполнять закон Ома, через лампочку к задней стенке того же элемента питания. А уж внутри самой батарейки под действием волшебницы-химии заряды снова оказались в первоначальной точке своего похода. И поэтому там, где напряжение было 1,5 вольта, оно таким и остается. То есть, напряжение постоянно одно, а заряды непрерывно движутся и последовательно проходят лампочку и гальванический элемент.

И это обычно рисуют на схеме вот так:

По закону Ома I=U/R

Тогда сопротивление лампочки (с тем током и напряжением, которые я написал) получится

R = 1/U , где R = 1 Ом

А мощность будет выделяться P = I * U , то есть P=2,25 Вm

В последовательной цепи, особенно на таком простом и несомненном примере, видно, что ток, который бежит по ней от начала до конца, — все время один и тот же. А если мы теперь возьмем две лампочки и сделаем так, чтобы ток пробегал сначала по одной, а потом по другой, то будет опять то же самое — ток будет и в той лампочке, и в другой снова одинаковым. Хотя другим по величине. Ток теперь испытывает сопротивление двух лампочек, но у каждой из них сопротивление как было, так и осталось, ведь оно определяется исключительно физическими свойствами самой лампочки. Новый ток вычисляем опять по закону Ома.

Он получится равным I=U/R+R,то есть 0,75А, ровно половина того тока, который был сначала.

В этом случае току приходится преодолевать уже два сопротивления, он становится меньше. Что и видно по свечению лампочек — они теперь горят вполнакала. А общее сопротивление цепочки из двух лампочек будет равно сумме их сопротивлений. Зная арифметику, можно в отдельном случае воспользоваться и действием умножения: если последовательно соединены N одинаковых лампочек, то общее их сопротивление будет равно N, умноженное на R, где R — сопротивление одной лампочки. Логика безупречная.

А мы продолжим наши опыты. Теперь сделаем нечто подобное, что мы провернули с лампочками, но только на левой стороне цепи: добавим еще один гальванический элемент, точно такой, как первый. Как видим, теперь у нас в два раза увеличилось общее напряжение, а ток стал снова 1,5 А, о чем и сигнализируют лампочки, загоревшись снова в полную силу.

Делаем вывод:

  • При последовательном соединении электрической цепи сопротивления и напряжения ее элементов суммируются, а ток на всех элементах остается неизменным.

Легко проверить, что это утверждение справедливо как для активных компонентов (гальванических элементов), так и для пассивных (лампочек, резисторов).

То есть это значит, что напряжение, измеренное на одном резисторе (оно называется падением напряжения), можно смело суммировать с напряжением, измеренным на другом резисторе, и в сумме получатся те же 3 В. А на каждом из сопротивлений оно окажется равным половине — то есть 1,5 В. И это справедливо. Два гальванических элемента вырабатывают свои напряжения, а две лампочки их потребляют. Потому что в источнике напряжения энергия химических процессов превращается в электроэнергию, принявшую вид напряжения, а в лампочках та же самая энергия из электрической превращается в тепловую и световую.

Вернемся к первой схеме, подключим в ней еще одну лампочку, но иначе.

Теперь напряжение в точках, соединяющих две ветки, то же, что и на гальваническом элементе — 1,5 В. Но так как сопротивление у обеих лампочек тоже такое, как и было, то и ток через каждую из них пойдет 1,5 А — ток «полного накала».

Гальванический элемент теперь питает их током одновременно, следовательно, из него вытекают сразу оба эти тока. То есть общий ток из источника напряжения будет равен 1,5 А + 1,5 А = 3,0 А.

В чем же отличие этой схемы от схемы, когда те же самые лампочки были включены последовательно? Только в накале лампочек, то есть только в токе.

Тогда ток был 0,75 А, а теперь он стал сразу 3 А.

Получается, если сравнить с первоначальной схемой, то при последовательном соединении лампочек (схема 2) току сопротивления оказывалось больше (отчего он уменьшался, и лампочки теряли светимость), а параллельное подключение оказывает МЕНЬШЕ сопротивления, хотя сопротивление лампочек осталось неизменным. В чем тут дело?

А дело в том, что мы забываем одну интересную истину, что всякая палка о двух концах.

Когда мы говорим, что резистор сопротивляется току, то как бы забываем, что он ток все-таки проводит. И теперь, когда подключили лампочки параллельно, увеличилось суммарное для них свойство проводить ток, а не сопротивляться ему. Ну и, соответственно, некую величину

G , по аналогии с сопротивлением R и следовало бы назвать проводимостью. И должна она в параллельном соединении проводников суммироваться.

Ну и вот она

Закон Ома тогда будет выглядеть

I = U * G &

И в случае параллельного соединения ток I будет равен U*(G+G) = 2*U*G, что мы как раз и наблюдаем.

Замена элементов цепи общим эквивалентным элементом

Инженерам часто приходится узнавать токи и напряжения во всех частях схем. А реальные электрические схемы бывают достаточно сложными и разветвленными и могут содержать множество элементов, активно потребляющих электроэнергию и соединенных друг с другом в совершенно разных сочетаниях. Это называется расчет электрических схем. Он делается при проектировании энергоснабжения домов, квартир, организаций. При этом очень важно, какие токи и напряжения будут действовать в электрической цепи, хотя бы для того, чтобы выбрать подходящие им сечения проводов, нагрузки на всю сеть или ее части, и так далее. А уж насколько сложны бывают электронные схемы, содержащие тысячи, а то и миллионы элементов, думаю, понятно всякому.

Самое первое что, напрашивается — это воспользоваться знанием того, как ведут себя токи напряжения в таких простейших соединениях сети, как последовательное и параллельное. Делают так: вместо найденного в сети последовательного соединения двух или более активных устройств-потребителей (как наши лампочки) нарисовать один, но чтобы его сопротивление было таким же, как у обоих. Тогда картина токов и напряжений в остальной части схемы не изменится. Аналогично и с параллельным соединением: вместо них нарисовать такой элемент, ПРОВОДИМОСТЬ которого была бы такой же, как у обоих.

Теперь если схему перерисовать, заменив последовательные и параллельные соединения одним элементом, то получим схему, которая называется «схемой эквивалентного замещения».

Такую процедуру можно продолжать до тех пор, пока у нас не останется наипростейшая — которой мы в самом начале иллюстрировали закон Ома. Только вместо лампочки будет стоять одно сопротивление, которое и называют эквивалентным сопротивлением нагрузки.

Это первая задача. Она дает нам возможность по закону Ома рассчитать общий ток во всей сети, или общий ток нагрузки.

Вот это и есть полный расчет электрической сети.

Примеры

Пусть цепь содержит 9 активных сопротивлений. Это могут быть лампочки или что-то другое.

На ее входные клеммы подано напряжение в 60 В.

Значения сопротивлений для всех элементов следующие:

Найти все неизвестные токи и напряжения.

Надо пойти по пути поиска параллельных и последовательных участков сети, рассчитывать эквивалентные им сопротивления и постепенно упрощать схему. Видим, что R 3 , R 9 и R 6 соединены последовательно. Тогда им эквивалентное сопротивление R э 3, 6, 9 будет равно их сумме R э 3, 6, 9 = 1 + 4 + 1 Ом = 6 Ом.

Теперь заменяем параллельный кусочек из сопротивлений R 8 и R э 3, 6, 9, получая R э 8, 3, 6, 9 . Только при параллельном соединении проводников, складывать придется проводимости.

Проводимость измеряется в единицах, называемых сименсами, обратных омам.

Если перевернуть дробь, получим сопротивление R э 8, 3, 6, 9 = 2 Ом

Совершенно так же, как в первом случае, объединяем сопротивления R 2 , R э 8, 3, 6, 9 и R 5, включенные последовательно, получая R э 2, 8, 3, 6, 9, 5 = 1 + 2 + 1 = 4 Ом.

Осталось два шага: получить сопротивление, эквивалентное двум резисторам параллельного соединения проводников R 7 и R э 2, 8, 3, 6, 9, 5.

Оно равно R э 7, 2, 8, 3, 6, 9, 5 = 1/(1/4+1/4)=1/(2/4)=4/2 = 2 Ом

На последнем шаге просуммируем все последовательно включенные сопротивления R 1 , R э 7, 2, 8, 3, 6, 9, 5 и R 4 и получим сопротивление, эквивалентное сопротивлению всей цепи R э и равное сумме этих трех сопротивлений

R э = R 1 + R э 7, 2, 8, 3, 6, 9, 5 + R4 = 1 + 2 + 1 = 4 Ом

Ну и вспомним, в честь кого назвали единицу сопротивлений, написанную нами в последней из этих формул, и вычислим по его закону общий ток во всей цепи I

Теперь, двигаясь в обратном направлении, в сторону все большего усложнения сети, можно получать по закону Ома токи и напряжения во всех цепочках нашей достаточно простой схемы.

Так обычно и рассчитывают схемы электроснабжения квартир, которые состоят из параллельных и последовательных участков. Что, как правило, не годится в электронике, потому что там многое по-другому устроено, и все гораздо замысловатее. И вот такую, например, схему, когда не поймешь, параллельное это соединение проводников или последовательное, рассчитывают по законам Кирхгофа.

Сопротивление проводников. Параллельное и последовательное соединение проводников.

Электри́ческое сопротивле́ние — физическая величина, характеризующая свойства проводника препятствовать прохождению электрического тока и равная отношениюнапряжения на концах проводника к силе тока, протекающего по нему . Сопротивление для цепей переменного тока и для переменных электромагнитных полей описывается понятиями импеданса и волнового сопротивления. Сопротивлением (резистором) также называют радиодеталь, предназначенную для введения в электрические цепи активного сопротивления.

Сопротивление (часто обозначается буквой R или r ) считается, в определённых пределах, постоянной величиной для данного проводника; её можно рассчитать как

R — сопротивление;

U — разность электрических потенциалов (напряжение) на концах проводника;

I — сила тока, протекающего между концами проводника под действием разности потенциалов.

При последовательном соединении проводников (рис. 1.9.1) сила тока во всех проводниках одинакова:

По закону Ома, напряжения U 1 и U 2 на проводниках равны

При последовательном соединении полное сопротивление цепи равно сумме сопротивлений отдельных проводников.

Этот результат справедлив для любого числа последовательно соединенных проводников.

При параллельном соединении (рис. 1.9.2) напряжения U 1 и U 2 на обоих проводниках одинаковы:

Этот результат следует из того, что в точках разветвления токов (узлы A и B ) в цепи постоянного тока не могут накапливаться заряды. Например, к узлу A за время Δt подтекает заряд I Δt , а утекает от узла за то же время заряд I 1 Δt + I 2 Δt . Следовательно,I = I 1 + I 2 .

Записывая на основании закона Ома

При параллельном соединении проводников величина, обратная общему сопротивлению цепи, равна сумме величин, обратных сопротивлениям параллельно включенных проводников.

Этот результат справедлив для любого числа параллельно включенных проводников.

Формулы для последовательного и параллельного соединения проводников позволяют во многих случаях рассчитывать сопротивление сложной цепи, состоящей из многих резисторов. На рис. 1.9.3 приведен пример такой сложной цепи и указана последовательность вычислений.

Следует отметить, что далеко не все сложные цепи, состоящие из проводников с различными сопротивлениями, могут быть рассчитаны с помощью формул для последовательного и параллельного соединения. На рис. 1.9.4 приведен пример электрической цепи, которую нельзя рассчитать указанным выше методом.

Последовательным называется такое соединение резисторов, когда конец одного проводника соединяется с началом другого и т.д. (рис. 1). При последовательном соединении сила тока на любом участке электрической цепи одинакова. Это объясняется тем, что заряды не могут накапливаться в узлах цепи. Их накопление привело бы к изменению напряженности электрического поля, а следовательно, и к изменению силы тока. Поэтому

Амперметр А измеряет силу тока в цепи и обладает малым внутренним сопротивлением (R A 0).

Включенные вольтметры V 1 и V 2 измеряют напряжение U 1 и U 2 на сопротивлениях R 1 и R 2 . Вольтметр V измеряет подведенное к клеммам М и N напряжение U. Вольтметры показывают, что при последовательном соединении напряжение U равно сумме напряжений на отдельных участках цепи:

Применяя закон Ома для каждого участка цепи, получим:

где R — общее сопротивление последовательно соединенной цепи. Подставляя U, U 1 , U 2 в формулу (1), имеем

Сопротивление цепи, состоящей из n последовательно соединенных резисторов, равно сумме сопротивлений этих резисторов:

Если сопротивления отдельных резисторов равны между собой, т.е. R 1 = R 2 = … = R n , то общее сопротивление этих резисторов при последовательном соединении в n раз больше сопротивления одного резистора: R = nR 1 .

При последовательном соединении резисторов справедливо соотношение

т.е. напряжения на резисторах прямо пропорциональны сопротивлениям.

Параллельным называется такое соединение резисторов, когда одни концы всех резисторов соединены в один узел, другие концы — в другой узел (рис. 2). Узлом называется точка разветвленной цепи, в которой сходятся более двух проводников. При параллельном соединении резисторов к точкам М и N подключен вольтметр. Он показывает, что напряжения на отдельных участках цепи с сопротивлениями R 1 и R 2 равны. Это объясняется тем, что работа сил стационарного электрического поля не зависит от формы траектории:

Амперметр показывает, что сила тока I в неразветвленной части цепи равна сумме сил токов I 1 и I 2 в параллельно соединенных проводниках R 1 и R 2:

Это вытекает и из закона сохранения электрического заряда. Применим закон Ома для отдельных участков цепи и всей цепи с общим сопротивлением R:

Подставляя I, I 1 и I 2 в формулу (2), получим.

Последовательным называется такое соединение резисторов, когда конец одного проводника соединяется с началом другого и т.д. (рис. 1). При последовательном соединении сила тока на любом участке электрической цепи одинакова. Это объясняется тем, что заряды не могут накапливаться в узлах цепи. Их накопление привело бы к изменению напряженности электрического поля, а следовательно, и к изменению силы тока. Поэтому

\(~I = I_1 = I_2 .\)

Амперметр А измеряет силу тока в цепи и обладает малым внутренним сопротивлением (R A → 0).

Включенные вольтметры V 1 и V 2 измеряют напряжение U 1 и U 2 на сопротивлениях R 1 и R 2 . Вольтметр V измеряет подведенное к клеммам Μ и N напряжение U . Вольтметры показывают, что при последовательном соединении напряжение U равно сумме напряжений на отдельных участках цепи:

\(~U = U_1 + U_2 . \qquad (1)\)

Применяя закон Ома для каждого участка цепи, получим:

\(~U = IR ; \ U_1 = IR_1 ; \ U_2 = IR_2 ,\)

где R — общее сопротивление последовательно соединенной цепи.n R_i .\)

Если сопротивления отдельных резисторов равны между собой, т.е. R 1 = R 2 = … = R n , то общее сопротивление этих резисторов при последовательном соединении в n раз больше сопротивления одного резистора: R = nR 1 .

При последовательном соединении резисторов справедливо соотношение \(~\frac{U_1}{U_2} = \frac{R_1}{R_2}\), т.е. напряжения на резисторах прямо пропорциональны сопротивлениям.

Параллельным называется такое соединение резисторов, когда одни концы всех резисторов соединены в один узел, другие концы — в другой узел (рис. 2). Узлом называется точка разветвленной цепи, в которой сходятся более двух проводников. При параллельном соединении резисторов к точкам Μ и N подключен вольтметр. Он показывает, что напряжения на отдельных участках цепи с сопротивлениями R 1 и R 2 равны. Это объясняется тем, что работа сил стационарного электрического поля не зависит от формы траектории:

\(~U = U_1 = U_2 .n \frac{1}{R_i} .\)

Если сопротивления всех n параллельно соединенных резисторов одинаковы и равны R 1 то \(~\frac 1R = \frac{n}{R_1}\) . Откуда \(~R = \frac{R_1}{n}\) .

Сопротивление цепи, состоящей из n одинаковых параллельно соединенных резисторов, в n раз меньше сопротивления каждого из них.

При параллельном соединении резисторов справедливо соотношение \(~\frac{I_1}{I_2} = \frac{R_2}{R_1}\), т.е. силы токов в ветвях параллельно соединенной цепи обратно пропорциональны сопротивлениям ветвей.

Литература

Аксенович Л. А. Физика в средней школе: Теория. Задания. Тесты: Учеб. пособие для учреждений, обеспечивающих получение общ. сред, образования / Л. А. Аксенович, Н.Н.Ракина, К. С. Фарино; Под ред. К. С. Фарино. — Мн.: Адукацыя i выхаванне, 2004. — C. 257-259.

Темы кодификатора ЕГЭ : параллельное и последовательное соединение проводников, смешанное соединение проводников.

Есть два основных способа соединения проводников друг с другом — это последовательное и параллельное соединения. Различные комбинации последовательного и параллельного соединений приводят к смешанному соединению проводников.

Мы будем изучать свойства этих соединений, но сначала нам понадобится некоторая вводная информация.

Проводник, обладающий сопротивлением , мы называем резистором и изображаем следующим образом (рис. 1 ):

Рис. 1. Резистор

Напряжение на резисторе — это разность потенциалов стационарного электрического поля между концами резистора. Между какими именно концами? В общем-то, это неважно, но обычно удобно согласовывать разность потенциалов с направлением тока.

Ток в цепи течёт от «плюса» источника к «минусу». В этом направлении потенциал стационарного поля убывает. Напомним ещё раз, почему это так.

Пусть положительный заряд перемещается по цепи из точки в точку , проходя через резистор (рис. 2 ):

Рис. 2.

Стационарное поле совершает при этом положительную работу .

Так как alt=»q > 0″> и alt=»A > 0″> , то и alt=»\varphi_a — \varphi_b > 0″> , т. е. alt=»\varphi_a > \varphi_b»> .

Поэтому напряжение на резисторе мы вычисляем как разность потенциалов в направлении тока: .

Сопротивление подводящих проводов обычно пренебрежимо мало; на электрических схемах оно считается равным нулю. Из закона Ома следует тогда, что потенциал не меняется вдоль провода: ведь если и , то . (рис. 3 ):

Рис. 3.

Таким образом, при рассмотрении электрических цепей мы пользуемся идеализацией, которая сильно упрощает их изучение. А именно, мы считаем, что потенциал стационарного поля изменяется лишь при переходе через отдельные элементы цепи, а вдоль каждого соединительного провода остаётся неизменным . В реальных цепях потенциал монотонно убывает при движении от положительной клеммы источника к отрицательной.

Последовательное соединение

При последовательном соединении проводников конец каждого проводника соединяется с началом следующего за ним проводника.

Рассмотрим два резистора и , соединённых последовательно и подключённых к источнику постоянного напряжения (рис. 4 ). Напомним, что положительная клемма источника обозначается более длинной чертой, так что ток в данной схеме течёт по часовой стрелке.

Рис. 4. Последовательное соединение

Сформулируем основные свойства последовательного соединения и проиллюстрируем их на этом простом примере.

1. При последовательном соединении проводников сила тока в них одинакова.
В самом деле, через любое поперечное сечение любого проводника за одну секунду будет проходить один и тот же заряд. Ведь заряды нигде не накапливаются, из цепи наружу не уходят и не поступают в цепь извне.

2. Напряжение на участке, состоящем из последовательно соединённых проводников, равно сумме напряжений на каждом проводнике .

Действительно, напряжение на участке — это работа поля по переносу единичного заряда из точки в точку ; напряжение на участке — это работа поля по переносу единичного заряда из точки в точку . Складываясь, эти две работы дадут работу поля по переносу единичного заряда из точки в точку , то есть напряжение на всём участке:

Можно и более формально, без всяких словесных объяснений:

3. Сопротивление участка, состоящего из последовательно соединённых проводников, равно сумме сопротивлений каждого проводника.

Пусть — сопротивление участка . По закону Ома имеем:

что и требовалось.

Можно дать интуитивно понятное объяснение правила сложения сопротивлений на одном частном примере. Пусть последовательно соединены два проводника из одинакового вещества и с одинаковой площадью поперечного сечения , но с разными длинами и .

Сопротивления проводников равны:

Эти два проводника образуют единый проводник длиной и сопротивлением

Но это, повторяем, лишь частный пример. Сопротивления будут складываться и в самом общем случае — если различны также вещества проводников и их поперечные сечения.
Доказательство этого даётся с помощью закона Ома, как показано выше.
Наши доказательства свойств последовательного соединения, приведённые для двух проводников, переносятся без существенных изменений на случай произвольного числа проводников.

Параллельное соединение

При параллельном соединении проводников их начала подсоединяются к одной точке цепи, а концы — к другой точке.

Снова рассматриваем два резистора, на сей раз соединённые параллельно (рис. 5 ).

Рис. 5. Параллельное соединение

Резисторы подсоединены к двум точкам: и . Эти точки называются узлами или точками разветвления цепи. Параллельные участки называются также ветвями ; участок от к (по направлению тока) называется неразветвлённой частью цепи.

Теперь сформулируем свойства параллельного соединения и докажем их для изображённого выше случая двух резисторов.

1. Напряжение на каждой ветви одинаково и равно напряжению на неразветвлённой части цепи.
В самом деле, оба напряжения и на резисторах и равны разности потенциалов между точками подключения:

Этот факт служит наиболее отчётливым проявлением потенциальности стационарного электрического поля движущихся зарядов.

2. Сила тока в неразветвлённой части цепи равна сумме сил токов в каждой ветви.
Пусть, например, в точку за время из неразветвлённого участка поступает заряд . За это же время из точки к резистору уходит заряд , а к резистору — заряд .

Ясно, что . В противном случае в точке накапливался бы заряд, меняя потенциал данной точки, что невозможно (ведь ток постоянный, поле движущихся зарядов стационарно, и потенциал каждой точки цепи не меняется со временем). Тогда имеем:

что и требовалось.

3. Величина, обратная сопротивлению участка параллельного соединения, равна сумме величин, обратных сопротивлениям ветвей.
Пусть — сопротивление разветвлённого участка . Напряжение на участке равно ; ток, текущий через этот участок, равен . Поэтому:

Сокращая на , получим:

(1)

что и требовалось.

Как и в случае последовательного соединения, можно дать объяснение данного правила на частном примере, не обращаясь к закону Ома.
Пусть параллельно соединены проводники из одного вещества с одинаковыми длинами , но разными поперечными сечениями и . Тогда это соединение можно рассматривать как проводник той же длины , но с площадью сечения . Имеем:

Приведённые доказательства свойств параллельного соединения без существенных изменений переносятся на случай любого числа проводников.

Из соотношения (1) можно найти :

(2)

К сожалению, в общем случае параллельно соединённых проводников компактного аналога формулы (2) не получается, и приходится довольствоваться соотношением

(3)

Тем не менее, один полезный вывод из формулы (3) сделать можно. Именно, пусть сопротивления всех резисторов одинаковы и равны . Тогда:

Мы видим, что сопротивление участка из параллельно соединённых одинаковых проводников в раз меньше сопротивления одного проводника.

Смешанное соединение

Смешанное сединение проводников, как следует из названия, может являться совокупностью любых комбинаций последовательного и параллельного соединений, причём в состав этих соединений могут входить как отдельные резисторы, так и более сложные составные участки.

Расчёт смешанного соединения опирается на уже известные свойства последовательного и параллельного соединений. Ничего нового тут уже нет: нужно только аккуратно расчленить данную схему на более простые участки, соединённые последовательно или параллельно.

Рассмотрим пример смешанного соединения проводников (рис. 6 ).

Рис. 6. Смешанное соединение

Пусть В, Ом, Ом, Ом, Ом, Ом. Найдём силу тока в цепи и в каждом из резисторов.

Наша цепь состоит из двух последовательно соединённых участков и . Сопротивление участка :

Ом.

Участок является параллельным соединением: два последовательно включённых резистора и подключены параллельно к резистору . Тогда:

Ом.

Сопротивление цепи:

Ом.

Теперь находим силу тока в цепи:

Для нахождения тока в каждом резисторе вычислим напряжения на обоих участках:

(Заметим попутно, что сумма этих напряжений равна В, т. е. напряжению в цепи, как и должно быть при последовательном соединении.)

Оба резистора и находятся под напряжением , поэтому:

(В сумме имеем А, как и должно быть при параллельном соединении.)

Сила тока в резисторах и одинакова, так как они соединены последовательно:

Стало быть, через резистор течёт ток A.

Падение напряжения при параллельном соединении. Соединения проводников

При решении задач принято преобразовывать схему, так, чтобы она была как можно проще. Для этого применяют эквивалентные преобразования. Эквивалентными называют такие преобразования части схемы электрической цепи, при которых токи и напряжения в не преобразованной её части остаются неизменными.

Существует четыре основных вида соединения проводников: последовательное, параллельное, смешанное и мостовое.

Последовательное соединение

Последовательное соединение – это такое соединение, при котором сила тока на всем участке цепи одинакова. Ярким примером последовательного соединения является старая елочная гирлянда. Там лампочки подключены последовательно, друг за другом. Теперь представьте, одна лампочка перегорает, цепь нарушена и остальные лампочки гаснут. Выход из строя одного элемента, ведет за собой отключение всех остальных, это является существенным недостатком последовательного соединения.

При последовательном соединении сопротивления элементов суммируются.

Параллельное соединение

Параллельное соединение – это соединение, при котором напряжение на концах участка цепи одинаково. Параллельное соединение наиболее распространено, в основном потому, что все элементы находятся под одним напряжением, сила тока распределена по-разному и при выходе одного из элементов все остальные продолжают свою работу.

При параллельном соединении эквивалентное сопротивление находится как:

В случае двух параллельно соединенных резисторов

В случае трех параллельно подключенных резисторов:

Смешанное соединение

Смешанное соединение – соединение, которое является совокупностью последовательных и параллельных соединений. Для нахождения эквивалентного сопротивления нужно, “свернуть” схему поочередным преобразованием параллельных и последовательных участков цепи.


Сначала найдем эквивалентное сопротивление для параллельного участка цепи, а затем прибавим к нему оставшееся сопротивление R 3 . Следует понимать, что после преобразования эквивалентное сопротивление R 1 R 2 и резистор R 3 , соединены последовательно.

Итак, остается самое интересное и самое сложное соединение проводников.

Мостовая схема

Мостовая схема соединения представлена на рисунке ниже.



Для того чтобы свернуть мостовую схему, один из треугольников моста, заменяют эквивалентной звездой.

И находят сопротивления R 1 , R 2 и R 3 .

Последовательное, параллельное и смешанное соединения резисторов. Значительное число приемников, включенных в электрическую цепь (электрические лампы, электронагревательные приборы и др.), можно рассматривать как некоторые элементы, имеющие определенное сопротивление. Это обстоятельство дает нам возможность при составлении и изучении электрических схем заменять конкретные приемники резисторами с определенными сопротивлениями. Различают следующие способы соединения резисторов (приемников электрической энергии): последовательное, параллельное и смешанное.

Последовательное соединение резисторов . При последовательном соединении нескольких резисторов конец первого резистора соединяют с началом второго, конец второго — с началом третьего и т. д. При таком соединении по всем элементам последовательной цепи проходит
один и тот же ток I.
Последовательное соединение приемников поясняет рис. 25, а.
.Заменяя лампы резисторами с сопротивлениями R1, R2 и R3, получим схему, показанную на рис. 25, б.
Если принять, что в источнике Ro = 0, то для трех последовательно соединенных резисторов согласно второму закону Кирхгофа можно написать:

E = IR 1 + IR 2 + IR 3 = I(R 1 + R 2 + R 3) = IR эк (19)

где R эк = R 1 + R 2 + R 3 .
Следовательно, эквивалентное сопротивление последовательной цепи равно сумме сопротивлений всех последовательно соединенных резисторов.Так как напряжения на отдельных участках цепи согласно закону Ома: U 1 =IR 1 ; U 2 = IR 2 , U 3 = IR з и в данном случае E = U, то длярассматриваемой цепи

U = U 1 + U 2 +U 3 (20)

Следовательно, напряжение U на зажимах источника равно сумме напряжений на каждом из последовательно включенных резисторов.
Из указанных формул следует также, что напряжения распределяются между последовательно соединенными резисторами пропорционально их сопротивлениям:

U 1: U 2: U 3 = R 1: R 2: R 3 (21)

т. е. чем больше сопротивление какого-либо приемника в последовательной цепи, тем больше приложенное к нему напряжение.

В случае если последовательно соединяются несколько, например п, резисторов с одинаковым сопротивлением R1, эквивалентное сопротивление цепи Rэк будет в п раз больше сопротивления R1, т. е. Rэк = nR1. Напряжение U1 на каждом резисторе в этом случае в п раз меньше общего напряжения U:

При последовательном соединении приемников изменение сопротивления одного из них тотчас же влечет за собой изменение напряжения на других связанных с ним приемниках. При выключении или обрыве электрической цепи в одном из приемников и в остальных приемниках прекращается ток. Поэтому последовательное соединение приемников применяют редко — только в том случае, когда напряжение источника электрической энергии больше номинального напряжения, на которое рассчитан потребитель. Например, напряжение в электрической сети, от которой питаются вагоны метрополитена, составляет 825 В, номинальное же напряжение электрических ламп, применяемых в этих вагонах, 55 В. Поэтому в вагонах метрополитена электрические лампы включают последовательно по 15 ламп в каждой цепи.
Параллельное соединение резисторов . При параллельном соединении нескольких приемников они включаются между двумя точками электрической цепи, образуя параллельные ветви (рис. 26, а). Заменяя

лампы резисторами с сопротивлениями R1, R2, R3, получим схему, показанную на рис. 26, б.
При параллельном соединении ко всем резисторам приложено одинаковое напряжение U. Поэтому согласно закону Ома:

I 1 =U/R 1 ; I 2 =U/R 2 ; I 3 =U/R 3 .

Ток в неразветвленной части цепи согласно первому закону Кирхгофа I = I 1 +I 2 +I 3 , или

I = U / R 1 + U / R 2 + U / R 3 = U (1/R 1 + 1/R 2 + 1/R 3) = U / R эк (23)

Следовательно, эквивалентное сопротивление рассматриваемой цепи при параллельном соединении трех резисторов определяется формулой

1/R эк = 1/R 1 + 1/R 2 + 1/R 3 (24)

Вводя в формулу (24) вместо значений 1/R эк, 1/R 1 , 1/R 2 и 1/R 3 соответствующие проводимости G эк, G 1 , G 2 и G 3 , получим: эквивалентная проводимость параллельной цепи равна сумме проводимостей параллельно соединенных резисторов :

G эк = G 1 + G 2 +G 3 (25)

Таким образом, при увеличении числа параллельно включаемых резисторов результирующая проводимость электрической цепи увеличивается, а результирующее сопротивление уменьшается.
Из приведенных формул следует, что токи распределяются между параллельными ветвями обратно пропорционально их электрическим сопротивлениям или прямо пропорционально их проводимостям. Например, при трех ветвях

I 1: I 2: I 3 = 1/R 1: 1/R 2: 1/R 3 = G 1 + G 2 + G 3 (26)

В этом отношении имеет место полная аналогия между распределением токов по отдельным ветвям и распределением потоков воды по трубам.
Приведенные формулы дают возможность определить эквивалентное сопротивление цепи для различных конкретных случаев. Например, при двух параллельно включенных резисторах результирующее сопротивление цепи

R эк =R 1 R 2 /(R 1 +R 2)

при трех параллельно включенных резисторах

R эк =R 1 R 2 R 3 /(R 1 R 2 +R 2 R 3 +R 1 R 3)

При параллельном соединении нескольких, например n, резисторов с одинаковым сопротивлением R1 результирующее сопротивление цепи Rэк будет в n раз меньше сопротивления R1, т.е.

R эк = R1 / n (27)

Проходящий по каждой ветви ток I1, в этом случае будет в п раз меньше общего тока:

I1 = I / n (28)

При параллельном соединении приемников, все они находятся под одним и тем же напряжением, и режим работы каждого из них не зависит от остальных. Это означает, что ток, проходящий по какому-либо из приемников, не будет оказывать существенного влияния на другие приемники. При всяком выключении или выходе из строя любого приемника остальные приемники остаются вклю-

ченными. Поэтому параллельное соединение имеет существенные преимущества перед последовательным, вследствие чего оно получило наиболее широкое распространение. В частности, электрические лампы и двигатели, предназначенные для работы при определенном (номинальном) напряжении, всегда включают параллельно.
На электровозах постоянного тока и некоторых тепловозах тяговые двигатели в процессе регулирования скорости движения нужно включать под различные напряжения, поэтому они в процессе разгона переключаются с последовательного соединения на параллельное.

Смешанное соединение резисторов . Смешанным соединением называется такое соединение, при котором часть резисторов включается последовательно, а часть — параллельно. Например, в схеме рис. 27, а имеются два последовательно включенных резистора сопротивлениями R1 и R2, параллельно им включен резистор сопротивлением Rз, а резистор сопротивлением R4 включен последовательно с группой резисторов сопротивлениями R1, R2 и R3.
Эквивалентное сопротивление цепи при смешанном соединении обычно определяют методом преобразования, при котором сложную цепь последовательными этапами преобразовывают в простейшую. Например, для схемы рис. 27, а вначале определяют эквивалентное сопротивление R12 последовательно включенных резисторов с сопротивлениями R1 и R2: R12 = R1 + R2. При этом схема рис. 27, а заменяется эквивалентной схемой рис. 27, б. Затем определяют эквивалентное сопротивление R123 параллельно включенных сопротивлений и R3 по формуле

R 123 =R 12 R 3 /(R 12 +R 3)=(R 1 +R 2)R 3 /(R 1 +R 2 +R 3).

При этом схема рис. 27, б заменяется эквивалентной схемой рис. 27, в. После этого находят эквивалентное сопротивление всей цепи суммированием сопротивления R123 и последовательно включенного с ним сопротивления R4:

R эк = R 123 + R 4 = (R 1 + R 2) R 3 / (R 1 + R 2 + R 3) + R 4

Последовательное, параллельное и смешанное соединения широко применяют для изменения сопротивления пусковых реостатов при пуске э. п. с. постоянного тока.

При одновременном включении нескольких приемников электроэнергии в одну и ту же сеть, эти приемники можно легко рассматривать просто как элементы единой цепи, каждый из которых обладает собственным сопротивлением.

В ряде случаев такой подход оказывается вполне приемлемым: лампы накаливания, электрические обогреватели и т. п. — можно воспринимать как резисторы. То есть приборы можно заменить на их сопротивления, и легко произвести расчет параметров цепи.

Способ соединения приемников электроэнергии может быть одним из следующих: последовательный, параллельный или смешанный тип соединения.

Последовательное соединение

Когда несколько приемников (резисторов) соединяются в последовательную цепь, то есть второй вывод первого присоединяется к первому выводу второго, второй вывод второго соединяется с первым выводом третьего, второй вывод третьего с первым выводом четвертого и т. д., то при подключении такой цепи к источнику питания, через все элементы цепи потечет ток I одной и той же величины. Данную мысль поясняет приведенный рисунок.

Заменив приборы на их сопротивления, рисунок преобразуем в схему, тогда сопротивления с R1 по R4, соединенные последовательно, примут каждый на себя определенные напряжения, которые в сумме дадут значение ЭДС на зажимах источника питания. Для простоты здесь и далее изобразим источник в виде гальванического элемента.

Выразив падения напряжений через ток и через сопротивления, получим выражение для эквивалентного сопротивления последовательной цепи приемников: общее сопротивление последовательного соединения резисторов всегда равно алгебраической сумме всех сопротивлений, составляющих эту цепь. А поскольку напряжения на каждом из участков цепи можно найти из закона Ома (U = I*R, U1 = I*R1, U2 = I*R2 и т. д.) и E = U, то для нашей схемы получаем:

Напряжение на клеммах источника питания равно сумме падений напряжений на каждом из соединенных последовательно приемников, составляющих цепь.

Так как ток через всю цепь течет одного и того же значения, то справедливым будет утверждение, что напряжения на последовательно соединенных приемниках (резисторах) соотносятся между собой пропорционально сопротивлениям. И чем выше будет сопротивление, тем выше окажется и напряжение, приложенное к приемнику.

Для последовательного соединения резисторов в количестве n штук, обладающих одинаковыми сопротивлениями Rk, эквивалентное общее сопротивление цепи целиком будет в n раз больше каждого из этих сопротивлений: R = n*Rk. Соответственно и напряжения, приложенные к каждому из резисторов цепи будут между собой равны, и окажутся в n раз меньше напряжения, приложенного ко всей цепи: Uk = U/n.

Для последовательного соединения приемников электроэнергии характерны следующие свойства: если изменить сопротивление одного из приемников цепи, то напряжения на остальных приемниках цепи при этом изменятся; при обрыве одного из приемников ток прекратится во всей цепи, во всех остальных приемниках.

В силу этих особенностей последовательное соединение встречается редко, и используют его лишь там, где напряжение сети выше номинального напряжения приемников, в отсутствие альтернатив.

К примеру напряжением 220 вольт можно запитать две последовательно соединенные лампы равной мощности, каждая из которых рассчитана на напряжение 110 вольт. Ежели данные лампы при одинаковом номинальном напряжении питания будут обладать различной номинальной мощностью, то одна из них будет перегружена и скорее всего мгновенно перегорит.

Параллельное соединение

Параллельное соединение приемников предполагает включение каждого из них между парой точек электрической цепи с тем, чтобы они образовывали параллельные ветви, каждая из которых питается напряжением источника. Для наглядности опять заменим приемники их электрическими сопротивлениями, чтобы получить схему, по которой удобно вести расчет параметров.

Как уже было сказано, в случае параллельного соединения каждый из резисторов испытывает действие одного и того же напряжения. И в соответствии с законом Ома имеем: I1=U/R1, I2=U/R2, I3=U/R3.

Здесь I — ток источника. Первый закон Кирхгофа для данной цепи позволяет записать выражение для тока в неразветвленной ее части: I = I1+I2+I3.

Отсюда общее сопротивление для параллельного соединения между собой элементов цепи можно найти из формулы:

Величина обратная сопротивлению называется проводимостью G, и формулу для проводимости цепи, состоящей из нескольких параллельно соединенных элементов, также можно записать: G = G1 + G2 + G3. Проводимость цепи в случае параллельного соединения образующих ее резисторов равна алгебраической сумме проводимостей этих резисторов. Следовательно, при добавлении в цепь параллельных приемников (резисторов) суммарное сопротивление цепи уменьшится, а суммарная проводимость соответственно возрастет.

Токи в цепи состоящей из параллельно соединенных приемников, распределяются между ними прямо пропорционально их проводимостям, то есть обратно пропорционально их сопротивлениям. Здесь можно привести аналогию из гидравлики, где поток воды распределяется по трубам в соответствии с их сечениями, тогда большее сечение аналогично меньшему сопротивлению, то есть большей проводимости.

Если цепь состоит из нескольких (n) одинаковых резисторов, соединенных параллельно, то общее сопротивление цепи будет ниже в n раз, чем сопротивление одного из резисторов, а ток через каждый из резисторов будет меньше в n раз, чем общий ток: R = R1/n; I1 = I/n.

Цепь, состоящая из параллельно соединенных приемников, подключенная к источнику питания, отличается тем, что каждый из приемников находится под напряжением источника питания.

Для идеального источника электроэнергии справедливо утверждение: при подключении или отключении параллельно источнику резисторов, токи в остальных подключенных резисторах не изменятся, то есть при выходе из строя одного или нескольких приемников параллельной цепи, остальные будут продолжать работать в прежнем режиме.

В силу данных особенностей параллельное соединение обладает значительным преимуществом перед последовательным, и по этой причине именно соединение параллельное наиболее распространено в электрических сетях. Например, все электроприборы в наших домах предназначены для параллельного подключения к бытовой сети, и если отключить один, то остальным это ничуть не навредит.

Сравнение последовательных и параллельных цепей

Под смешанным соединением приемников понимают такое их соединение, когда часть или несколько из них соединены между собой последовательно, а другая часть или несколько — параллельно. При этом вся цепь может быть образована из разных соединений таких частей между собой. Для примера рассмотрим схему:

Три последовательно соединенных резистора подключены к источнику питания, параллельно одному из них подключены еще два, а третий — параллельно всей цепи. Для нахождения полного сопротивления цепи идут путем последовательных преобразований: сложную цепь последовательно приводят к простому виду, последовательно вычисляя сопротивление каждого звена, и так находят общее эквивалентное сопротивление.

Для нашего примера. Сначала находят общее сопротивление двух резисторов R4 и R5, соединенных последовательно, затем сопротивление параллельного соединения их с R2, потом прибавляют к полученному значению R1 и R3, и после — вычисляют значение сопротивления всей цепи, включая параллельную ветвь R6.

Различные способы соединения приемников электроэнергии применяют на практике для различных целей, чтобы решать конкретные поставленные задачи. Например, смешанное соединение можно встретить в схемах плавного заряда в мощных блоках питания, где нагрузка (конденсаторы после диодного моста) сначала получает питание последовательно через резистор, затем резистор шунтируется контактами реле, и нагрузка оказывается подключенной к диодному мосту параллельно.

Андрей Повный

Содержание:

Во всех электрических схемах используются резисторы, представляющие собой элементы, с точно установленным значением сопротивления. Благодаря специфическим качествам этих устройств, становится возможной регулировка напряжения и силы тока на любых участках схемы. Данные свойства лежат в основе работы практически всех электронных приборов и оборудования. Так, напряжение при параллельном и последовательном соединении резисторов будет отличаться. Поэтому каждый вид соединения может применяться только в определенных условиях, чтобы та или иная электрическая схема могла в полном объеме выполнять свои функции.

Напряжение при последовательном соединении

При последовательном соединении два резистора и более соединяются в общую цепь таким образом, что каждый из них имеет контакт с другим устройством только в одной точке. Иначе говоря, конец первого резистора соединяется с началом второго, а конец второго — с началом третьего и т.д.

Особенностью данной схемы является прохождение через все подключенные резисторы одного и того же значения электрического тока. С возрастанием количества элементов на рассматриваемом участке цепи, течение электрического тока становится все более затрудненным. Это происходит из-за увеличения общего сопротивления резисторов при их последовательном соединении. Данное свойство отражается формулой: R общ = R 1 + R 2 .

Распределение напряжения, в соответствии с законом Ома, осуществляется на каждый резистор по формуле: V Rn = I Rn x R n . Таким образом, при увеличении сопротивления резистора, возрастает и падающее на него напряжение.

Напряжение при параллельном соединении

При параллельном соединении, включение резисторов в электрическую цепь выполняется таким образом, что все элементы сопротивлений подключаются друг к другу сразу обоими контактами. Одна точка, представляющая собой электрический узел, может соединять одновременно несколько резисторов.

Такое соединение предполагает течение отдельного тока в каждом резисторе. Сила этого тока находится в обратно пропорциональной . В результате, происходит увеличение общей проводимости данного участка цепи, при общем уменьшении сопротивления. В случае параллельного соединения резисторов с различным сопротивлением, значение общего сопротивления на этом участке всегда будет ниже самого маленького сопротивления отдельно взятого резистора.

На представленной схеме, напряжение между точками А и В представляет собой не только общее напряжение для всего участка, но и напряжение, поступающее к каждому отдельно взятому резистору. Таким образом, в случае параллельного соединения, напряжение, подаваемое ко всем резисторам, будет одинаковым.

В результате, напряжение при параллельном и последовательном соединении будет отличаться в каждом случае. Благодаря этому свойству, имеется реальная возможность отрегулировать данную величину на любом участке цепи.

Содержание:

В электрических цепях используются различные типы соединений. Основными являются последовательные, параллельные и смешанные схемы подключений. В первом случае используется несколько сопротивлений, соединенных в единую цепочку друг за другом. То есть, начало одного резистора соединяется с концом второго, а начало второго — с концом третьего и так далее, до любого количества сопротивлений. Сила тока при последовательном соединении будет одинаковой во всех точках и на всех участках. Для определения и сравнения других параметров электрической цепи, следует рассматривать и остальные виды соединений, обладающие собственными свойствами и характеристиками.

Последовательное и параллельное соединение сопротивлений

Любая нагрузка обладает сопротивлением, препятствующим свободному течению электрического тока. Его путь проходит от источника тока, через проводники к нагрузке. Для нормального прохождения тока, проводник должен обладать хорошей проводимостью и легко отдавать электроны. Это положение пригодится далее при рассмотрении вопроса, что такое последовательное соединение.

В большинстве электрических цепей применяются медные проводники. Каждая цепь содержит приемники энергии — нагрузки, обладающие различными сопротивлениями. Параметры соединения лучше всего рассматривать на примере внешней цепи источника тока, состоящей из трех резисторов R1, R2, R3. Последовательное соединение предполагает поочередное включение этих элементов в замкнутую цепь. То есть начало R1 соединяется с концом R2, а начало R2 — с концом R3 и так далее. В такой цепочке может быть любое количество резисторов. Эти символы используют в расчетах .

На всех участках будет одинаковой: I = I1 = I2 = I3, а общее сопротивление цепи составит сумму сопротивлений всех нагрузок: R = R1 + R2 + R3. Остается лишь определить, каким будет при последовательном соединении. В соответствии с законом Ома, напряжение представляет собой силу тока и сопротивления: U = IR. Отсюда следует, что напряжение на источнике тока будет равно сумме напряжений на каждой нагрузке, поскольку ток везде одинаковый: U = U1 + U2 + U3.

При постоянном значении напряжения, ток при последовательном соединении будет находиться в зависимости от сопротивления цепи. Поэтому при изменении сопротивления хотя-бы на одной из нагрузок, произойдет изменение сопротивления во всей цепи. Кроме того, изменятся ток и напряжение на каждой нагрузке. Основным недостатком последовательного соединения считается прекращение работы всех элементов цепи, при выходе из строя даже одного из них.

Совершенно другие характеристики тока, напряжения и сопротивления получаются при использовании параллельного соединения. В этом случае начала и концы нагрузок соединяются в двух общих точках. Происходит своеобразное разветвление тока, что приводит к снижению общего сопротивления и росту общей проводимости электрической цепи.

Для того чтобы отобразить эти свойства, вновь понадобится закон Ома. В данном случае сила тока при параллельном соединении и его формула будет выглядеть так: I = U/R. Таким образом, при параллельном соединении n-го количества одинаковых резисторов, общее сопротивление цепи будет в n раз меньше любого из них: Rобщ = R/n. Это указывает на обратно пропорциональное распределение токов в нагрузках по отношению к сопротивлениям этих нагрузок. То есть, при увеличении параллельно включенных сопротивлений, сила тока в них будет пропорционально уменьшаться. В виде формул все характеристики отображаются следующим образом: сила тока — I = I1 + I2 + I3, напряжение — U = U1 = U2 = U3, сопротивление — 1/R = 1/R1 + 1/R2 + 1/R3.

При неизменном значении напряжения между элементами, токи в этих резисторах не имеют зависимости друг от друга. Если один или несколько резисторов будут выключены из цепи, это никак не повлияет на работу других устройств, остающихся включенными. Данный фактор является основным преимуществом параллельного соединения электроприборов.

В схемах обычно не используется только последовательное соединение и параллельное соединение сопротивлений, они применяются в комбинированном виде, известном как . Для вычисления характеристик таких цепей применяются формулы обоих вариантов. Все расчеты разбиваются на несколько этапов, когда вначале определяются параметры отдельных участков, после чего они складываются и получается общий результат.

Законы последовательного и параллельного соединения проводников

Основным законом, применяемым при расчетах различных видов соединений, является закон Ома. Его основным положением является наличие на участке цепи силы тока, прямо пропорциональной напряжению и обратно пропорциональной сопротивлению на данном участке. В виде формулы этот закон выглядит так: I = U/R. Он служит основой для проведения расчетов электрических цепей, соединяемых последовательно или параллельно. Порядок вычислений и зависимость всех параметров от закона Ома наглядно показаны на рисунке. Отсюда выводится и формула последовательного соединения.

Более сложные вычисления с участием других величин требуют применения . Его основное положение заключается в том, что несколько последовательно соединенных источников тока, будут обладать электродвижущей силой (ЭДС), составляющей алгебраическую сумму ЭДС каждого из них. Общее сопротивление этих батарей будет состоять из суммы сопротивлений каждой батареи. Если выполняется параллельное подключение n-го количества источников с равными ЭДС и внутренними сопротивлениями, то общая сумма ЭДС будет равно ЭДС на любом из источников. Значение внутреннего сопротивления составит rв = r/n. Эти положения актуальны не только для источников тока, но и для проводников, в том числе и формулы параллельное соединение проводников.

В том случае, когда ЭДС источников будет иметь разное значение, для расчетов силы тока на различных участках цепи применяются дополнительные правила Кирхгофа.

Формула расчета напряжения при последовательном соединении. Виды соединения проводников

Проверим справедливость показанных здесь формул на простом эксперименте.

Возьмём два резистора МЛТ-2 на 3 и 47 Ом и соединим их последовательно. Затем измерим общее сопротивление получившейся цепи цифровым мультиметром. Как видим оно равно сумме сопротивлений резисторов, входящих в эту цепочку.


Замер общего сопротивления при последовательном соединении

Теперь соединим наши резисторы параллельно и замерим их общее сопротивление.


Измерение сопротивления при параллельном соединении

Как видим, результирующее сопротивление (2,9 Ом) меньше самого меньшего (3 Ом), входящего в цепочку. Отсюда вытекает ещё одно известное правило, которое можно применять на практике:

При параллельном соединении резисторов общее сопротивление цепи будет меньше наименьшего сопротивления, входящего в эту цепь.

Что ещё нужно учитывать при соединении резисторов?

Во-первых, обязательно учитывается их номинальная мощность. Например, нам нужно подобрать замену резистору на 100 Ом и мощностью 1 Вт . Возьмём два резистора по 50 Ом каждый и соединим их последовательно. На какую мощность рассеяния должны быть рассчитаны эти два резистора?

Поскольку через последовательно соединённые резисторы течёт один и тот же постоянный ток (допустим 0,1 А ), а сопротивление каждого из них равно 50 Ом , тогда мощность рассеивания каждого из них должна быть не менее 0,5 Вт . В результате на каждом из них выделится по 0,5 Вт мощности. В сумме это и будет тот самый 1 Вт .

Данный пример достаточно грубоват. Поэтому, если есть сомнения, стоит брать резисторы с запасом по мощности.

Подробнее о мощности рассеивания резистора читайте .

Во-вторых, при соединении стоит использовать однотипные резисторы, например, серии МЛТ. Конечно, нет ничего плохого в том, чтобы брать разные. Это лишь рекомендация.

Содержание:

Течение тока в электрической цепи осуществляется по проводникам, в направлении от источника к потребителям. В большинстве подобных схем используются медные провода и электрические приемники в заданном количестве, обладающие различным сопротивлением. В зависимости выполняемых задач, в электрических цепях используется последовательное и параллельное соединение проводников. В некоторых случаях могут быть применены оба типа соединений, тогда этот вариант будет называться смешанным. Каждая схема имеет свои особенности и отличия, поэтому их нужно обязательно заранее учитывать при проектировании цепей, ремонте и обслуживании электрооборудования.

Последовательное соединение проводников

В электротехнике большое значение имеет последовательное и параллельное соединение проводников в электрической цепи. Среди них часто используется схема последовательного соединения проводников предполагающая такое же соединение потребителей. В этом случае включение в цепь выполняется друг за другом в порядке очередности. То есть, начало одного потребителя соединяется с концом другого при помощи проводов, без каких-либо ответвлений.

Свойства такой электрической цепи можно рассмотреть на примере участков цепи с двумя нагрузками. Силу тока, напряжение и сопротивление на каждом из них следует обозначить соответственно, как I1, U1, R1 и I2, U2, R2. В результате, получились соотношения, выражающие зависимость между величинами следующим образом: I = I1 = I2, U = U1 + U2, R = R1 + R2. Полученные данные подтверждаются практическим путем с помощью проведения измерений амперметром и вольтметром соответствующих участков.

Таким образом, последовательное соединение проводников отличается следующими индивидуальными особенностями:

  • Сила тока на всех участках цепи будет одинаковой.
  • Общее напряжение цепи составляет сумму напряжений на каждом участке.
  • Общее сопротивление включает в себя сопротивления каждого отдельного проводника.

Данные соотношения подходят для любого количества проводников, соединенных последовательно. Значение общего сопротивления всегда выше, чем сопротивление любого отдельно взятого проводника. Это связано с увеличением их общей длины при последовательном соединении, что приводит и к росту сопротивления.

Если соединить последовательно одинаковые элементы в количестве n, то получится R = n х R1, где R — общее сопротивление, R1 — сопротивление одного элемента, а n — количество элементов. Напряжение U, наоборот, делится на равные части, каждая из которых в n раз меньше общего значения. Например, если в сеть с напряжением 220 вольт последовательно включаются 10 ламп одинаковой мощности, то напряжение в любой из них составит: U1 = U/10 = 22 вольта.

Проводники, соединенные последовательно, имеют характерную отличительную особенность. Если во время работы отказал хотя-бы один из них, то течение тока прекращается во всей цепи. Наиболее ярким примером является , когда одна перегоревшая лампочка в последовательной цепи, приводит к выходу из строя всей системы. Для установления перегоревшей лампочки понадобится проверка всей гирлянды.

Параллельное соединение проводников

В электрических сетях проводники могут соединяться различными способами: последовательно, параллельно и комбинированно. Среди них параллельное соединение это такой вариант, когда проводники в начальных и конечных точках соединяются между собой. Таким образом, начала и концы нагрузок соединяются вместе, а сами нагрузки располагаются параллельно относительно друг друга. В электрической цепи могут содержаться два, три и более проводников, соединенных параллельно.

Если рассматривать последовательное и параллельное соединение, сила тока в последнем варианте может быть исследована с помощью следующей схемы. Берутся две лампы накаливания, обладающие одинаковым сопротивлением и соединенные параллельно. Для контроля к каждой лампочке подключается собственный . Кроме того, используется еще один амперметр, контролирующий общую силу тока в цепи. Проверочная схема дополняется источником питания и ключом.

После замыкания ключа нужно контролировать показания измерительных приборов. Амперметр на лампе № 1 покажет силу тока I1, а на лампе № 2 — силу тока I2. Общий амперметр показывает значение силы тока, равное сумме токов отдельно взятых, параллельно соединенных цепей: I = I1 + I2. В отличие от последовательного соединения, при перегорании одной из лампочек, другая будет нормально функционировать. Поэтому в домашних электрических сетях используется параллельное подключение приборов.

С помощью такой же схемы можно установить значение эквивалентного сопротивления. С этой целью в электрическую цепь добавляется вольтметр. Это позволяет измерить напряжение при параллельном соединении, сила тока при этом остается такой же. Здесь также имеются точки пересечения проводников, соединяющих обе лампы.

В результате измерений общее напряжение при параллельном соединении составит: U = U1 = U2. После этого можно рассчитать эквивалентное сопротивление, условно заменяющее все элементы, находящиеся в данной цепи. При параллельном соединении, в соответствии с законом Ома I = U/R, получается следующая формула: U/R = U1/R1 + U2/R2, в которой R является эквивалентным сопротивлением, R1 и R2 — сопротивления обеих лампочек, U = U1 = U2 — значение напряжения, показываемое вольтметром.

Следует учитывать и тот фактор, что токи в каждой цепи, в сумме составляют общую силу тока всей цепи. В окончательном виде формула, отражающая эквивалентное сопротивление будет выглядеть следующим образом: 1/R = 1/R1 + 1/R2. При увеличении количества элементов в таких цепях — увеличивается и число слагаемых в формуле. Различие в основных параметрах отличают друг от друга и источников тока, позволяя использовать их в различных электрических схемах.

Параллельное соединение проводников характеризуется достаточно малым значением эквивалентного сопротивления, поэтому сила тока будет сравнительно высокой. Данный фактор следует учитывать, когда в розетки включается большое количество электроприборов. В этом случае сила тока значительно возрастает, приводя к перегреву кабельных линий и последующим возгораниям.

Законы последовательного и параллельного соединения проводников

Данные законы, касающиеся обоих видов соединений проводников, частично уже были рассмотрены ранее.

Для более четкого их понимания и восприятия в практической плоскости, последовательное и параллельное соединение проводников, формулы следует рассматривать в определенной последовательности:

  • Последовательное соединение предполагает одинаковую силу тока в каждом проводнике: I = I1 = I2.
  • параллельное и последовательное соединение проводников объясняет в каждом случае по-своему. Например, при последовательном соединении, напряжения на всех проводниках будут равны между собой: U1 = IR1, U2 = IR2. Кроме того, при последовательном соединении напряжение составляет сумму напряжений каждого проводника: U = U1 + U2 = I(R1 + R2) = IR.
  • Полное сопротивление цепи при последовательном соединении состоит из суммы сопротивлений всех отдельно взятых проводников, независимо от их количества.
  • При параллельном соединении напряжение всей цепи равно напряжению на каждом из проводников: U1 = U2 = U.
  • Общая сила тока, измеренная во всей цепи, равна сумме токов, протекающих по всем проводникам, соединенных параллельно между собой: I = I1 + I2.

Для того чтобы более эффективно проектировать электрические сети, нужно хорошо знать последовательное и параллельное соединение проводников и его законы, находя им наиболее рациональное практическое применение.

Смешанное соединение проводников

В электрических сетях как правило используется последовательное параллельное и смешанное соединение проводников, предназначенное для конкретных условий эксплуатации. Однако чаще всего предпочтение отдается третьему варианту, представляющему собой совокупность комбинаций, состоящих из различных типов соединений.

В таких смешанных схемах активно применяется последовательное и параллельное соединение проводников, плюсы и минусы которых обязательно учитываются при проектировании электрических сетей. Эти соединения состоят не только из отдельно взятых резисторов, но и довольно сложных участков, включающих в себя множество элементов.

Смешанное соединение рассчитывается в соответствии с известными свойствами последовательного и параллельного соединения. Метод расчета заключается в разбивке схемы на более простые составные части, которые считаются отдельно, а потом суммируются друг с другом.

Во многих электрических схемах мы можем обнаружить последовательное и . Разработчик схем может, например, объединить несколько резисторов со стандартными значениями (E-серии), чтобы получить необходимое сопротивление.

Последовательное соединении резисторов — это такое соединение, при котором ток, протекающий через каждый резистор одинаков, поскольку имеется только одно направление для протекания тока. В тоже время падение напряжения будет пропорционально сопротивлению каждого резистора в последовательной цепи.

Последовательное соединение резисторов

Пример № 1

Используя закон Ома, необходимо вычислить эквивалентное сопротивление серии последовательно соединенных резисторов (R1. R2, R3), а так же падение напряжения и мощность для каждого резистора:

Все данные могут быть получены с помощью закона Ома и для лучшего понимания представлены в виде следующей таблицы:

Пример № 2

а) без подключенного резистора R3

б) с подключенным резистором R3

Как вы можете видеть, выходное напряжение U без нагрузочного резистора R3, составляет 6 вольт, но то же выходное напряжение при подключении R3 становится всего лишь 4 В. Таким образом, нагрузка, подключенная к делителю напряжения, провоцирует дополнительное падение напряжение. Данный эффект снижения напряжения может быть компенсирован с помощью установленного вместо постоянного резистора, с помощью которого можно скорректировать напряжение на нагрузке.

Онлайн калькулятор расчета сопротивления последовательно соединенных резисторов

Чтобы быстро вычислить общее сопротивление двух и более резисторов, соединенных последовательно, вы можете воспользоваться следующим онлайн калькулятором:

Подведем итог

Когда два или несколько резисторов соединены вместе (вывод одного соединяется с выводом другого резистора) — то это последовательное соединение резисторов. Ток, протекающий через резисторы имеет одно и тоже значение, но падение напряжения на них не одно и то же. Оно определяется сопротивлением каждого резистора, которое рассчитывается по закону Ома (U = I * R).

Последовательное соединение сопротивлений

Возьмем три неизменных сопротивления R1, R2 и R3 и включим их в цепь так, чтоб конец первого сопротивления R1 был соединен с началом второго сопротивления R 2, конец второго — с началом третьего R 3, а к началу первого сопротивления и к концу третьего подведем проводники от источника тока (рис. 1 ).

Такое соединение сопротивлений именуется поочередным. Разумеется, что ток в таковой цепи будет во всех ее точках один и тот же.

Рис 1 . Последовательное соединение сопротивлений

Как найти общее сопротивление цепи, если все включенные в нее поочередно сопротивления мы уже знаем? Используя положение, что напряжение U на зажимах источника тока равно сумме падений напряжений на участках цепи, мы можем написать:

U = U1 + U2 + U3

где

U1 = IR1 U2 = IR2 и U3 = IR3

либо

IR = IR1 + IR2 + IR3

Вынеся в правой части равенства I за скобки, получим IR = I(R1 + R2 + R3) .

Поделив сейчас обе части равенства на I , будем совсем иметь R = R1 + R2 + R3

Таким макаром, мы сделали вывод, что при поочередном соединении сопротивлений общее сопротивление всей цепи равно сумме сопротивлений отдельных участков.

Проверим этот вывод на последующем примере. Возьмем три неизменных сопротивления, величины которых известны (к примеру, R1 == 10 Ом, R 2 = 20 Ом и R 3 = 50 Ом). Соединим их поочередно (рис. 2 ) и подключим к источнику тока, ЭДС которого равна 60 В (внутренним сопротивлением источника тока пренебрегаем).

Рис. 2. Пример поочередного соединения 3-х сопротивлений

Подсчитаем, какие показания должны дать приборы, включенные, как показано на схеме, если замкнуть цепь. Определим наружное сопротивление цепи: R = 10 + 20 + 50 = 80 Ом.

Найдем ток в цепи по закону Ома: 60 / 80 = 0 ,75 А

Зная ток в цепи и сопротивления ее участков, определим падение напряжения на каждое участке цепи U 1 = 0,75х 10 = 7,5 В, U 2 = 0,75 х 20=15 В, U3 = 0,75 х 50 = 37,5 В.

Зная падение напряжений на участках, определим общее падение напряжения во наружной цепи, т. е. напряжение на зажимах источника тока U = 7,5+15 + 37,5 = 60 В.

Мы получили таким макаром, что U = 60 В, т. е. несуществующее равенство ЭДС источника тока и его напряжения. Разъясняется это тем, что мы пренебрегли внутренним сопротивлением источника тока.

Замкнув сейчас ключ выключатель К, можно убедиться по устройствам, что наши подсчеты приблизительно верны.

Возьмем два неизменных сопротивления R1 и R2 и соединим их так, чтоб начала этих сопротивлений были включены в одну общую точку а, а концы — в другую общую точку б. Соединив потом точки а и б с источником тока, получим замкнутую электронную цепь. Такое соединение сопротивлений именуется параллельным соединением.

Рис 3. Параллельное соединение сопротивлений

Проследим течение тока в этой цепи. От положительного полюса источника тока по соединительному проводнику ток дойдет до точки а. В точке а он разветвится, потому что тут сама цепь разветвляется на две отдельные ветки: первую ветвь с сопротивлением R1 и вторую — с сопротивлением R2. Обозначим токи в этих ветвях соответственно через I1 и I 2. Любой из этих токов пойдет по собственной ветки до точки б. В этой точке произойдет слияние токов в один общий ток, который и придет к отрицательному полюсу источника тока.

Таким макаром, при параллельном соединении сопротивлений выходит разветвленная цепь. Поглядим, какое же будет соотношение меж токами в составленной нами цепи.

Включим амперметр меж положительным полюсом источника тока (+) и точкой а и заметим его показания. Включив потом амперметр (показанный «а рисунке пунктиром) в провод, соединяющий точку б с отрицательным полюсом источника тока (-), заметим, что прибор покажет ту же величину силы тока.

Означает, сила тока в цепи до ее разветвления (до точки а) равна силе тока после разветвления цепи (после точки б).

Будем сейчас включать амперметр попеременно в каждую ветвь цепи, запоминая показания прибора. Пусть в первой ветки амперметр покажет силу тока I1 , а во 2-ой — I 2. Сложив эти два показания амперметра, мы получим суммарный ток, по величине равный току I до разветвления (до точки а).

Как следует, сила тока, протекающего до точки разветвления, равна сумме сил токов, утекающих от этой точки. I = I1 + I2 Выражая это формулой, получим

Это соотношение, имеющее огромное практическое значение, носит заглавие закона разветвленной цепи .

Разглядим сейчас, каково будет соотношение меж токами в ветвях.

Включим меж точками а и б вольтметр и поглядим, что он нам покажет. Во-1-х, вольтметр покажет напряжение источника тока, потому что он подключен, как это видно из рис. 3 , конкретно к зажимам источника тока. Во-2-х, вольтметр покажет падения напряжений U1 и U2 на сопротивлениях R1 и R2, потому что он соединен с началом и концом каждого сопротивления.

Как следует, при параллельном соединении сопротивлений напряжение на зажимах источника тока равно падению напряжения на каждом сопротивлении.

Это дает нам право написать, что U = U1 = U2 ,

где U — напряжение на зажимах источника тока; U1 — падение напряжения на сопротивлении R1 , U2 — падение напряжения на сопротивлении R2. Вспомним, что падение напряжения на участке цепи численно равно произведению силы тока, протекающего через этот участок, на сопротивление участка U = IR .

Потому для каждой ветки можно написать: U1 = I1R1 и U2 = I2R2 , но потому что U1 = U2, то и I1R1 = I2R2 .

Применяя к этому выражению правило пропорции, получим I1/ I2 = U2 / U1 т. е. ток в первой ветки будет во столько раз больше (либо меньше) тока во 2-ой ветки, во сколько раз сопротивление первой ветки меньше (либо больше) сопротивления 2-ой ветки.

Итак, мы пришли к принципиальному выводу, заключающемуся в том, что при параллельном соединении сопротивлений общий ток цепи разветвляется на токи, назад пропорциональные величинам сопротивлении параллельных веток. По другому говоря, чем больше сопротивление ветки, тем наименьший ток потечет через нее, и, напротив, чем меньше сопротивление ветки, тем больший ток потечет через эту ветвь.

Убедимся в корректности этой зависимости на последующем примере. Соберем схему, состоящую из 2-ух параллельно соединенных сопротивлений R1 и R 2, присоединенных к источнику тока. Пусть R1 = 10 Ом, R2 = 20 Ом и U = 3 В.

Подсчитаем поначалу, что покажет нам амперметр, включенный в каждую ветвь:

I1 = U / R1 = 3 / 10 = 0 ,3 А = 300 мА

I 2 = U / R 2 = 3 / 20 = 0,15 А = 150 мА

Общий ток в цепи I = I1 +I2 = 300 + 150 = 450 мА

Проделанный нами расчет подтверждает, что при параллельном соединении сопротивлений ток в цепи разветвляется назад пропорционально сопротивлениям.

Вправду, R1 == 10 Ом в два раза меньше R 2 = 20 Ом, при всем этом I1 = 300 мА в два раза больше I2 = 150 мА. Общий ток в цепи I = 450 мА разветвился на две части так, что большая его часть (I1 = 300 мА) пошла через наименьшее сопротивление (R1 = 10 Ом), а наименьшая часть (R2 = 150 мА) -через большее сопротивление (R 2 = 20 Ом).

Такое разветвление тока в параллельных ветвях сходно с течением воды по трубам. Представьте для себя трубу А, которая в каком-то месте разветвляется на две трубы Б и В различного поперечника (рис. 4). Потому что поперечник трубы Б больше поперечника трубок В, то через трубу Б в одно и то же время пройдет больше воды, чем через трубу В, которая оказывает сгустку воды большее сопротивление.

Рис. 4

Разглядим сейчас, чему будет равно общее сопротивление наружной цепи, состоящей из 2-ух параллельно соединенных сопротивлений.

Под этим общим сопротивлением наружной цепи нужно осознавать такое сопротивление, которым можно было бы поменять при данном напряжении цепи оба параллельно включенных сопротивления, не изменяя при всем этом тока до разветвления. Такое сопротивление именуется эквивалентным сопротивлением.

Вернемся к цепи, показанной на рис. 3, и поглядим, чему будет равно эквивалентное сопротивление 2-ух параллельно соединенных сопротивлений. Применяя к этой цепи закон Ома, мы можем написать: I = U/R , где I — ток во наружной цепи (до точки разветвления), U — напряжение наружной цепи, R — сопротивление наружной цепи, т. е. эквивалентное сопротивление.

Точно так же для каждой ветки I1 = U1 / R1 , I2 = U2 / R2 , где I1 и I 2 — токи в ветвях; U1 и U2 — напряжение на ветвях; R1 и R2 — сопротивления веток.

По закону разветвленной цепи: I = I1 + I2

Подставляя значения токов, получим U / R = U1 / R1 + U2 / R2

Потому что при параллельном соединении U = U1 = U2 , то можем написать U / R = U / R1 + U / R2

Вынеся U в правой части равенства за скобки, получим U / R = U (1 / R1 + 1 / R2 )

Разделив сейчас обе части равенства на U , будем совсем иметь 1 / R = 1 / R1 + 1 / R2

Помня, что проводимостью именуется величина, оборотная сопротивлению , мы можем сказать, что в приобретенной формуле 1 / R — проводимость наружной цепи; 1 / R1 проводимость первой ветки; 1 / R2- проводимость 2-ой ветки.

На основании этой формулы делаем вывод: при параллельном соединении проводимость наружной цепи равна сумме проводимостей отдельных веток.

Как следует, чтоб найти эквивалентное сопротивление включенных параллельно сопротивлений, нужно найти проводимость цепи и взять величину, ей оборотную.

Из формулы также следует, что проводимость цепи больше проводимости каждой ветки, а это означает, что эквивалентное сопротивление наружной цепи меньше меньшего из включенных параллельно сопротивлений.

Рассматривая случай параллельного соединения сопротивлений, мы взяли более ординарную цепь, состоящую из 2-ух веток. Но на практике могут повстречаться случаи, когда цепь состоит из 3-х и поболее параллельных веток. Как поступать в этих случаях?

Оказывается, все приобретенные нами соотношения остаются справедливыми и для цепи, состоящей из хоть какого числа параллельно соединенных сопротивлений.

Чтоб убедиться в этом, разглядим последующий пример.

Возьмем три сопротивления R1 = 10 Ом, R2 = 20 Ом и R3 = 60 Ом и соединим их параллельно. Определим эквивалентное сопротивление цепи (рис. 5 ). R = 1 / 6 Как следует, эквивалентное сопротивление R = 6 Ом.

Таким макаром, эквивалентное сопротивление меньше меньшего из включенных параллельно в цепь сопротивлений , т. е. меньше сопротивления R1.

Поглядим сейчас, вправду ли это сопротивление является эквивалентным, т. е. таким, которое могло бы поменять включенные параллельно сопротивления в 10, 20 и 60 Ом, не изменяя при всем этом силы тока до разветвления цепи.

Допустим, что напряжение наружной цепи, а как следует, и напряжение на сопротивлениях R1, R2, R3 равно 12 В. Тогда сила токов в ветвях будет: I1 = U/R1 = 12 / 10 = 1 ,2 А I 2 = U/R 2 = 12 / 20 = 1 ,6 А I 3 = U/R1 = 12 / 60 = 0,2 А

Общий ток в цепи получим, пользуясь формулой I = I1 + I2 + I3 =1,2 + 0,6 + 0,2 = 2 А.

Проверим по формуле закона Ома, получится ли в цепи ток силой 2 А, если заместо 3-х параллельно включенных узнаваемых нам сопротивлений включено одно эквивалентное им сопротивление 6 Ом.

I = U / R = 12 / 6 = 2 А

Как лицезреем, отысканное нами сопротивление R = 6 Ом вправду является для данной цепи эквивалентным.

В этом можно убедиться и на измерительных устройствах, если собрать схему с взятыми нами сопротивлениями, измерить ток во наружной цепи (до разветвления), потом поменять параллельно включенные сопротивления одним сопротивлением 6 Ом и опять измерить ток. Показания амперметра и в том и в другом случае будут приблизительно схожими.

На практике могут повстречаться также параллельные соединения, для которых высчитать эквивалентное сопротивление можно проще, т. е. не определяя за ранее проводимостей, сходу отыскать сопротивление.

К примеру, если соединены параллельно два сопротивления R1 и R2 , то формулу 1 / R = 1 / R1 + 1 / R2 можно конвертировать так: 1/R = (R2 + R1) / R1 R2 и, решая равенство относительно R, получить R = R1 х R2 / (R1 + R2 ), т. е. при параллельном соединении 2-ух сопротивлений эквивалентное сопротивление цепи равно произведению включенных параллельно сопротивлений, деленному на их сумму.

Обычно все затрудняются ответить. А вот загадка эта в применении к электричеству решается вполне определенно.

Электричество начинается с закона Ома.

А уж если рассматривать дилемму в контексте параллельного или последовательного соединений — считая одно соединение курицей, а другое — яйцом, то сомнений вообще нет никаких.

Потому что закон Ома — это и есть самая первоначальная электрическая цепь. И она может быть только последовательной.

Да, придумали гальванический элемент и не знали, что с ним делать, поэтому сразу придумали еще лампочку. И вот что из этого получилось. Здесь напряжение в 1,5 В немедленно потекло в качестве тока, чтобы неукоснительно выполнять закон Ома, через лампочку к задней стенке того же элемента питания. А уж внутри самой батарейки под действием волшебницы-химии заряды снова оказались в первоначальной точке своего похода. И поэтому там, где напряжение было 1,5 вольта, оно таким и остается. То есть, напряжение постоянно одно, а заряды непрерывно движутся и последовательно проходят лампочку и гальванический элемент.

И это обычно рисуют на схеме вот так:

По закону Ома I=U/R

Тогда сопротивление лампочки (с тем током и напряжением, которые я написал) получится

R = 1/U , где R = 1 Ом

А мощность будет выделяться P = I * U , то есть P=2,25 Вm

В последовательной цепи, особенно на таком простом и несомненном примере, видно, что ток, который бежит по ней от начала до конца, — все время один и тот же. А если мы теперь возьмем две лампочки и сделаем так, чтобы ток пробегал сначала по одной, а потом по другой, то будет опять то же самое — ток будет и в той лампочке, и в другой снова одинаковым. Хотя другим по величине. Ток теперь испытывает сопротивление двух лампочек, но у каждой из них сопротивление как было, так и осталось, ведь оно определяется исключительно физическими свойствами самой лампочки. Новый ток вычисляем опять по закону Ома.

Он получится равным I=U/R+R,то есть 0,75А, ровно половина того тока, который был сначала.

В этом случае току приходится преодолевать уже два сопротивления, он становится меньше. Что и видно по свечению лампочек — они теперь горят вполнакала. А общее сопротивление цепочки из двух лампочек будет равно сумме их сопротивлений. Зная арифметику, можно в отдельном случае воспользоваться и действием умножения: если последовательно соединены N одинаковых лампочек, то общее их сопротивление будет равно N, умноженное на R, где R — сопротивление одной лампочки. Логика безупречная.

А мы продолжим наши опыты. Теперь сделаем нечто подобное, что мы провернули с лампочками, но только на левой стороне цепи: добавим еще один гальванический элемент, точно такой, как первый. Как видим, теперь у нас в два раза увеличилось общее напряжение, а ток стал снова 1,5 А, о чем и сигнализируют лампочки, загоревшись снова в полную силу.

Делаем вывод:

  • При последовательном соединении электрической цепи сопротивления и напряжения ее элементов суммируются, а ток на всех элементах остается неизменным.

Легко проверить, что это утверждение справедливо как для активных компонентов (гальванических элементов), так и для пассивных (лампочек, резисторов).

То есть это значит, что напряжение, измеренное на одном резисторе (оно называется падением напряжения), можно смело суммировать с напряжением, измеренным на другом резисторе, и в сумме получатся те же 3 В. А на каждом из сопротивлений оно окажется равным половине — то есть 1,5 В. И это справедливо. Два гальванических элемента вырабатывают свои напряжения, а две лампочки их потребляют. Потому что в источнике напряжения энергия химических процессов превращается в электроэнергию, принявшую вид напряжения, а в лампочках та же самая энергия из электрической превращается в тепловую и световую.

Вернемся к первой схеме, подключим в ней еще одну лампочку, но иначе.

Теперь напряжение в точках, соединяющих две ветки, то же, что и на гальваническом элементе — 1,5 В. Но так как сопротивление у обеих лампочек тоже такое, как и было, то и ток через каждую из них пойдет 1,5 А — ток «полного накала».

Гальванический элемент теперь питает их током одновременно, следовательно, из него вытекают сразу оба эти тока. То есть общий ток из источника напряжения будет равен 1,5 А + 1,5 А = 3,0 А.

В чем же отличие этой схемы от схемы, когда те же самые лампочки были включены последовательно? Только в накале лампочек, то есть только в токе.

Тогда ток был 0,75 А, а теперь он стал сразу 3 А.

Получается, если сравнить с первоначальной схемой, то при последовательном соединении лампочек (схема 2) току сопротивления оказывалось больше (отчего он уменьшался, и лампочки теряли светимость), а параллельное подключение оказывает МЕНЬШЕ сопротивления, хотя сопротивление лампочек осталось неизменным. В чем тут дело?

А дело в том, что мы забываем одну интересную истину, что всякая палка о двух концах.

Когда мы говорим, что резистор сопротивляется току, то как бы забываем, что он ток все-таки проводит. И теперь, когда подключили лампочки параллельно, увеличилось суммарное для них свойство проводить ток, а не сопротивляться ему. Ну и, соответственно, некую величину G , по аналогии с сопротивлением R и следовало бы назвать проводимостью. И должна она в параллельном соединении проводников суммироваться.

Ну и вот она

Закон Ома тогда будет выглядеть

I = U * G &

И в случае параллельного соединения ток I будет равен U*(G+G) = 2*U*G, что мы как раз и наблюдаем.

Замена элементов цепи общим эквивалентным элементом

Инженерам часто приходится узнавать токи и напряжения во всех частях схем. А реальные электрические схемы бывают достаточно сложными и разветвленными и могут содержать множество элементов, активно потребляющих электроэнергию и соединенных друг с другом в совершенно разных сочетаниях. Это называется расчет электрических схем. Он делается при проектировании энергоснабжения домов, квартир, организаций. При этом очень важно, какие токи и напряжения будут действовать в электрической цепи, хотя бы для того, чтобы выбрать подходящие им сечения проводов, нагрузки на всю сеть или ее части, и так далее. А уж насколько сложны бывают электронные схемы, содержащие тысячи, а то и миллионы элементов, думаю, понятно всякому.

Самое первое что, напрашивается — это воспользоваться знанием того, как ведут себя токи напряжения в таких простейших соединениях сети, как последовательное и параллельное. Делают так: вместо найденного в сети последовательного соединения двух или более активных устройств-потребителей (как наши лампочки) нарисовать один, но чтобы его сопротивление было таким же, как у обоих. Тогда картина токов и напряжений в остальной части схемы не изменится. Аналогично и с параллельным соединением: вместо них нарисовать такой элемент, ПРОВОДИМОСТЬ которого была бы такой же, как у обоих.

Теперь если схему перерисовать, заменив последовательные и параллельные соединения одним элементом, то получим схему, которая называется «схемой эквивалентного замещения».

Такую процедуру можно продолжать до тех пор, пока у нас не останется наипростейшая — которой мы в самом начале иллюстрировали закон Ома. Только вместо лампочки будет стоять одно сопротивление, которое и называют эквивалентным сопротивлением нагрузки.

Это первая задача. Она дает нам возможность по закону Ома рассчитать общий ток во всей сети, или общий ток нагрузки.

Вот это и есть полный расчет электрической сети.

Примеры

Пусть цепь содержит 9 активных сопротивлений. Это могут быть лампочки или что-то другое.

На ее входные клеммы подано напряжение в 60 В.

Значения сопротивлений для всех элементов следующие:

Найти все неизвестные токи и напряжения.

Надо пойти по пути поиска параллельных и последовательных участков сети, рассчитывать эквивалентные им сопротивления и постепенно упрощать схему. Видим, что R 3 , R 9 и R 6 соединены последовательно. Тогда им эквивалентное сопротивление R э 3, 6, 9 будет равно их сумме R э 3, 6, 9 = 1 + 4 + 1 Ом = 6 Ом.

Теперь заменяем параллельный кусочек из сопротивлений R 8 и R э 3, 6, 9, получая R э 8, 3, 6, 9 . Только при параллельном соединении проводников, складывать придется проводимости.

Проводимость измеряется в единицах, называемых сименсами, обратных омам.

Если перевернуть дробь, получим сопротивление R э 8, 3, 6, 9 = 2 Ом

Совершенно так же, как в первом случае, объединяем сопротивления R 2 , R э 8, 3, 6, 9 и R 5, включенные последовательно, получая R э 2, 8, 3, 6, 9, 5 = 1 + 2 + 1 = 4 Ом.

Осталось два шага: получить сопротивление, эквивалентное двум резисторам параллельного соединения проводников R 7 и R э 2, 8, 3, 6, 9, 5.

Оно равно R э 7, 2, 8, 3, 6, 9, 5 = 1/(1/4+1/4)=1/(2/4)=4/2 = 2 Ом

На последнем шаге просуммируем все последовательно включенные сопротивления R 1 , R э 7, 2, 8, 3, 6, 9, 5 и R 4 и получим сопротивление, эквивалентное сопротивлению всей цепи R э и равное сумме этих трех сопротивлений

R э = R 1 + R э 7, 2, 8, 3, 6, 9, 5 + R4 = 1 + 2 + 1 = 4 Ом

Ну и вспомним, в честь кого назвали единицу сопротивлений, написанную нами в последней из этих формул, и вычислим по его закону общий ток во всей цепи I

Теперь, двигаясь в обратном направлении, в сторону все большего усложнения сети, можно получать по закону Ома токи и напряжения во всех цепочках нашей достаточно простой схемы.

Так обычно и рассчитывают схемы электроснабжения квартир, которые состоят из параллельных и последовательных участков. Что, как правило, не годится в электронике, потому что там многое по-другому устроено, и все гораздо замысловатее. И вот такую, например, схему, когда не поймешь, параллельное это соединение проводников или последовательное, рассчитывают по законам Кирхгофа.

Расчет напряжения при параллельном соединении. Преимущества и недостатки параллельного и последовательного соединения лампочек

Последовательное и параллельное соединение проводников это основные виды соединения проводников, встречающиеся на практике. Так как электрические цепи, как правило, не состоят из однородных проводников одинакового сечения. Как же найти сопротивление цепи, если известны сопротивления ее отдельных частей.

Рассмотрим два типичных случая. Первый из них это когда два или боле проводников обладающих сопротивлением включены последовательно. Последовательно значит, что конец первого проводника подключен к началу второго и так далее. При таком включении проводников сила тока в каждом из них будет одинакова. А вот напряжение на каждом из них будет различным.

Рисунок 1 — последовательное соединение проводников

Падение напряжения на сопротивлениях можно определить исходя из закона Ома.

Формула 1 — Падение напряжения на сопротивлении

Сумма этих напряжений будет равна полному напряжению, приложенному к цепи. Напряжение на проводниках будет распределяться пропорционально их сопротивлению. То есть можно записать.

Формула 2 — соотношение между сопротивлением и напряжением

Суммарное же сопротивление цепи будет равно сумме всех сопротивлений включенных последовательно.

Формула 3 — вычисление суммарного сопротивления при параллельном включении

Второй случай, когда сопротивления в цепи включены параллельно друг другу. То есть в цепи есть два узла и все проводники обладающие сопротивлением подключаются к этим узлам. В такой цепи токи во всех ветвях в общем случае не равны друг другу. Но сумма всех токов в цепи после разветвления будет равна току до разветвления.

Рисунок 2 — Параллельное соединение проводников

Формула 4 — соотношение между токами в параллельных ветвях

Сила тока в каждой из разветвлённой цепи также подчиняется закону Ома. Напряжение на всех проводниках будет одинаково. Но сила тока будет разлучаться. В цепи, состоящей из параллельно соединенных проводников, токи распределяются пропорционально сопротивлениям.

Формула 5 — Распределение токов в параллельных ветвях

Чтобы найти полное сопротивление цепи в этом случае необходимо сложить величины обратные сопротивлениям то есть проводимости.

Формула 6 — Сопротивление параллельно включённых проводников

Также существует упрощённая формула для частного случая когда параллельно включены два одинаковых сопротивления.

1. Находят эквивалентное сопротивление участков цепи с параллельным соединением резисторов. Рисунок 2. Последовательное соединение резисторов. Для расчета сопротивления таких соединений, всю цепь разбивают на простейшие участки, из параллельно или последовательно соединенных резисторов.

Этот результат следует из того, что в точках разветвления токов (узлы A и B) в цепи постоянного тока не могут накапливаться заряды. Этот результат справедлив для любого числа параллельно включенных проводников.

На рис. 1.9.3 приведен пример такой сложной цепи и указана последовательность вычислений. Следует отметить, что далеко не все сложные цепи, состоящие из проводников с различными сопротивлениями, могут быть рассчитаны с помощью формул для последовательного и параллельного соединения.

При последовательном соединении проводников сила тока во всех проводниках одинакова. При параллельном соединении падение напряжения между двумя узлами, объединяющими элементы цепи, одинаково для всех элементов.

Т.е чем большее сопротивление резистора, тем большее напряжение на него падает. В результате к одной точке (электрическому узлу) может быть присоединено несколько резисторов. При таком соединении, через каждый резистор потечет отдельный ток. Сила данного тока будет обратно пропорциональна сопротивлению резистора.

Таким образом, при параллельном подсоединении резисторов с разным сопротивлением, общее сопротивление будет всегда меньше значения самого маленького отдельного резистора. Напряжение между точками A и B является как общим напряжением для всего участка цепи, так и напряжением, падающим на каждый резистор в отдельности. Смешанным соединением называют участок цепи, где часть резисторов соединяются между собой последовательно, а часть параллельно.

Цепь разбивают на участки с только пареллельным или только последовательным соединением. Вычисляют общее сопротивление для каждого отдельного участка. Вычисляют общее сопротивление для всей цепи смешанного соединения. Также существует более быстрый способ расчета общего сопротивления для смешанного соединения. Если резисторы соединяются последоватеьно — складывать.

То есть при последовательном соединении резисторы подключатся друг за другом. На рисунке 4 показан простейший пример смешанного соединения резисторов. После расчета эквивалентных сопротивлений резисторов перерисовывают схему. Обычно получается цепь из последовательно соединенных эквивалентных сопротивлений.4. Рисунок 5. Расчет сопротивления участка цепи при смешанном соединении резисторов.

В результате вы научитесь с нуля не тольно разрабатывать собственные устройства, но и сопрягать с ними различную переферию! Узел — точка разветвления цепи, в которой соединяются не менее трёх проводников. Последовательное соединение резисторов применяется для увеличения сопротивления.

Напряжение при параллельном соединении

Как видно, вычислить сопротивление двух параллельных резисторов значительно удобнее. Параллельное соединение резисторов часто используют в случаях, когда необходимо сопротивление с большей мощностью. Для этого, как правило, используют резисторы с одинаковой мощностью и одинаковым сопротивлением.

Общее сопротивление Rобщ

Такое соединение сопротивлений называется последовательным. Мы получили таким образом, что U = 60 В, т. е. несуществующее равенство ЭДС источника тока и его напряжения. Будем теперь включать амперметр поочередно в каждую ветвь цепи, запоминая показания прибора. Следовательно, при параллельном соединении сопротивлений напряжение на зажимах источника тока равно падению напряжения на каждом сопротивлении.

Такое разветвление тока в параллельных ветвях сходно с течением жидкости по трубам. Рассмотрим теперь, чему будет равно общее сопротивление внешней цепи, состоящей из двух параллельно соединенных сопротивлений.

Вернемся к цепи, показанной на рис. 3, и посмотрим, чему будет равно эквивалентное сопротивление двух параллельно соединенных сопротивлений. Точно так же для каждой ветви I1 = U1 / R1, I2 = U2 / R2, где I1 и I2 — токи в ветвях; U1 и U2 — напряжение на ветвях; R1 и R2 — сопротивления ветвей.

Это значит, что общее сопротивление цепи всегда будет ниже любого параллельно включенного резистора. 2. Если эти участки включают последовательно соединенные резисторы, то сначала вычисляют их сопротивление. Применяя закон Ома для участка цепи, можно доказать, что полное сопротивление при последовательном соединении равно сумме сопротивлений отдельных проводников.

Содержание:

Как известно, соединение любого элемента схемы, независимо от его назначения, может быть двух видов — параллельное подключение и последовательное. Также возможно и смешанное, то есть последовательно параллельное соединение. Все зависит от назначения компонента и выполняемой им функции. А значит, и резисторы не избежали этих правил. Последовательное и параллельное сопротивление резисторов это по сути то же самое, что и параллельное и последовательное подключение источников света. В параллельной цепи схема подключения подразумевает вход на все резисторы из одной точки, а выход из другой. Попробуем разобраться, каким образом выполняется последовательное соединение, а каким — параллельное. И главное, в чем состоит разница между подобными соединениями и в каких случаях необходимо последовательное, а в каких параллельное соединение. Также интересен и расчет таких параметров, как общее напряжение и общее сопротивление цепи в случаях последовательного либо параллельного соединения. Начать следует с определений и правил.

Способы подключения и их особенности

Виды соединения потребителей или элементов играют очень важную роль, ведь именно от этого зависят характеристики всей схемы, параметры отдельных цепей и тому подобное. Для начала попробуем разобраться с последовательным подключением элементов к схеме.

Последовательное соединение

Последовательное подключение — это такое соединение, где резисторы (равно, как и другие потребители или элементы схем) подключаются друг за другом, при этом выход предыдущего подключается на вход следующего. Подобный вид коммутации элементов дает показатель, равный сумме сопротивлений этих элементов схемы. То есть если r1 = 4 Ом, а r2 = 6 Ом, то при подключении их в последовательную цепь, общее сопротивление составит 10 Ом. Если мы добавим последовательно еще один резистор на 5 Ом, сложение этих цифр даст 15 Ом — это и будет общее сопротивление последовательной цепи. То есть общие значения равны сумме всех сопротивлений. При его расчете для элементов, которые подключены последовательно, никаких вопросов не возникает — все просто и ясно. Именно поэтому не стоит даже останавливаться более серьезно на этой.

Совершенно по другим формулам и правилам производится расчет общего сопротивления резисторов при параллельном подключении, вот на нем имеет смысл остановиться поподробнее.

Параллельное соединение

Параллельным называется соединение, при котором все входы резисторов объединены в одной точке, а все выходы — во второй. Здесь главное понять, что общее сопротивление при подобном подключении будет всегда ниже, чем тот же параметр резистора, имеющего наименьшее.

Имеет смысл разобрать подобную особенность на примере, тогда понять это будет намного проще. Существует два резистора по 16 Ом, но при этом для правильного монтажа схемы требуется лишь 8 Ом. В данном случае при задействовании их обеих, при их параллельном включении в схему, как раз и получатся необходимые 8 Ом. Попробуем понять, по какой формуле возможны вычисления. Рассчитать этот параметр можно так: 1/Rобщ = 1/R1+1/R2, причем при добавлении элементов сумма может продолжаться до бесконечности.

Попробуем еще один пример. Параллельно соединены 2 резистора, с сопротивлением 4 и 10 Ом. Тогда общее будет равно 1/4 + 1/10, что будет равным 1:(0.25 + 0.1) = 1:0.35 = 2.85 Ом. Как видим, хотя резисторы и имели значительное сопротивление, при подключении их параллельнообщий показатель стал намного ниже.

Так же можно рассчитать общее сопротивление четырех параллельно подключенных резисторов, с номиналом 4, 5, 2 и 10 Ом. Вычисления, согласно формуле, будут такими: 1/Rобщ = 1/4+1/5+1/2+1/10, что будет равным 1:(0.25+0.2+0.5+0.1)=1/1.5 = 0.7 Ом.

Что же касается тока, протекающего через параллельно соединенные резисторы, то здесь необходимо обратиться к закону Кирхгофа, который гласит «сила тока при параллельном соединении, выходящего из цепи, равна току, входящему в цепь». А потому здесь законы физики решают все за нас. При этом общие показатели тока разделяются на значения, которые являются обратно пропорциональными сопротивлению ветки. Если сказать проще, то чем больше показатель сопротивления, тем меньшие токи будут проходить через этот резистор, но в общем, все же ток входа будет и на выходе. При параллельном соединении напряжение также остается на выходе таким же, как и на входе. Схема параллельного соединения указана ниже.

Последовательно-параллельное соединение

Последовательно-параллельное соединение — это когда схема последовательного соединения содержит в себе параллельные сопротивления. В таком случае общее последовательное сопротивление будет равно сумме отдельно взятых общих параллельных. Метод вычислений одинаковый в соответствующих случаях.

Подведем итог

Подводя итог всему вышеизложенному можно сделать следующие выводы:

  1. При последовательном соединении резисторов не требуется особых формул для расчета общего сопротивления. Необходимо лишь сложить все показатели резисторов — сумма и будет общим сопротивлением.
  2. При параллельном соединении резисторов, общее сопротивление высчитывается по формуле 1/Rобщ = 1/R1+1/R2…+Rn.
  3. Эквивалентное сопротивление при параллельном соединении всегда меньше минимального подобного показателя одного из резисторов, входящих в схему.
  4. Ток, равно как и напряжение в параллельном соединении остается неизменным, то есть напряжение при последовательном соединении равно как на входе, так и на выходе.
  5. Последовательно-параллельное соединение при подсчетах подчиняется тем же законам.

В любом случае, каким бы ни было подключение, необходимо четко рассчитывать все показатели элементов, ведь параметры имеют очень важную роль при монтаже схем. И если ошибиться в них, то либо схема не будет работать, либо ее элементы просто сгорят от перегрузки. По сути, это правило применимо к любым схемам, даже в электромонтаже. Ведь провод по сечению подбирают также исходя из мощности и напряжения. А если поставить лампочку номиналом в 110 вольт в цепь с напряжением 220, несложно понять, что она моментально сгорит. Так же и с элементами радиоэлектроники. А потому — внимательность и скрупулезность в расчетах — залог правильной работы схемы.

Содержание:

Все известные виды проводников обладают определенными свойствами, в том числе и электрическим сопротивлением. Это качество нашло свое применение в резисторах, представляющих собой элементы цепи с точно установленным сопротивлением. Они позволяют выполнять регулировку тока и напряжения с высокой точностью в схемах. Все подобные сопротивления имеют свои индивидуальные качества. Например, мощность при паралл ельном и последовательном соединении резисторов будет различной. Поэтому на практике очень часто используются различные методики расчетов, благодаря которым возможно получение точных результатов.

Свойства и технические характеристики резисторов

Как уже отмечалось, резисторы в электрических цепях и схемах выполняют регулировочную функцию. С этой целью используется закон Ома, выраженный формулой: I = U/R. Таким образом, с уменьшением сопротивления происходит заметное возрастание тока. И, наоборот, чем выше сопротивление, тем меньше ток. Благодаря этому свойству, резисторы нашли широкое применение в электротехнике. На этой основе создаются делители тока, использующиеся в конструкциях электротехнических устройств.

Помимо функции регулировки тока, резисторы применяются в схемах делителей напряжения. В этом случае закон Ома будет выглядеть несколько иначе: U = I x R. Это означает, что с ростом сопротивления происходит увеличение напряжения. На этом принципе строится вся работа устройств, предназначенных для деления напряжения. Для делителей тока используется паралл ельное соединение резисторов, а для — последовательное.

На схемах резисторы отображаются в виде прямоугольника, размером 10х4 мм. Для обозначения применяется символ R, который может быть дополнен значением мощности данного элемента. При мощности свыше 2 Вт, обозначение выполняется с помощью римских цифр. Соответствующая надпись наносится на схеме возле значка резистора. Мощность также входит в состав , нанесенной на корпус элемента. Единицами измерения сопротивления служат ом (1 Ом), килоом (1000 Ом) и мегаом (1000000 Ом). Ассортимент резисторов находится в пределах от долей ома до нескольких сотен мегаом. Современные технологии позволяют изготавливать данные элементы с довольно точными значениями сопротивления.

Важным параметром резистора считается отклонение сопротивления. Его измерение осуществляется в процентах от номинала. Стандартный ряд отклонений представляет собой значения в виде: + 20, + 10, + 5, + 2, + 1% и так далее до величины + 0,001%.

Большое значение имеет мощность резистора. По каждому из них во время работы проходит электрический ток, вызывающий нагрев. Если допустимое значение рассеиваемой мощности превысит норму, это приведет к выходу из строя резистора. Следует учитывать, что в процессе нагревания происходит изменение сопротивления элемента. Поэтому если устройства работают в широких диапазонах температур, применяется специальная величина, именуемая температурным коэффициентом сопротивления.

Для соединения резисторов в схемах используются три разных способа подключения — паралл ельное, последовательное и смешанное. Каждый способ обладает индивидуальными качествами, что позволяет применять данные элементы в самых разных целях.

Мощность при последовательном соединение

При соединение резисторов последовательно электрический ток по очереди проходит через каждое сопротивление. Значение тока в любой точке цепи будет одинаковым. Данный факт определяется с помощью закона Ома. Если сложить все сопротивления, приведенные на схеме, то получится следующий результат: R = 200+100+51+39 = 390 Ом.

Учитывая напряжение в цепи, равное 100 В, сила тока будет составлять I = U/R = 100/390 = 0,256 A.На основании полученных данных можно рассчитать мощность резисторов при последовательном соединении по следующей формуле: P = I 2 x R = 0,256 2 x 390 = 25,55 Вт.

  • P 1 = I 2 x R 1 = 0,256 2 x 200 = 13,11 Вт;
  • P 2 = I 2 x R 2 = 0,256 2 x 100 = 6,55 Вт;
  • P 3 = I 2 x R 3 = 0,256 2 x 51 = 3,34 Вт;
  • P 4 = I 2 x R 4 = 0,256 2 x 39 = 2,55 Вт.

Если сложить полученные мощность, то полная Р составит: Р = 13,11+6,55+3,34+2,55 = 25,55 Вт.

Мощность при паралл ельном соединение

При паралл ельном подключении все начала резисторов соединяются с одним узлом схемы, а концы — с другим. В этом случае происходит разветвление тока, и он начинает протекать по каждому элементу. В соответствии с законом Ома, сила тока будет обратно пропорциональна всем подключенным сопротивлениям, а значение напряжения на всех резисторах будет одним и тем же.

Прежде чем вычислять силу тока, необходимо выполнить расчет полной проводимости всех резисторов, применяя следующую формулу:

  • 1/R = 1/R 1 +1/R 2 +1/R 3 +1/R 4 = 1/200+1/100+1/51+1/39 = 0,005+0,01+0,0196+0,0256 = 0,06024 1/Ом.
  • Поскольку сопротивление является величиной, обратно пропорциональной проводимости, его значение составит: R = 1/0,06024 = 16,6 Ом.
  • Используя значение напряжения в 100 В, по закону Ома рассчитывается сила тока: I = U/R = 100 x 0,06024 = 6,024 A.
  • Зная силу тока, мощность резисторов, соединенных паралл ельно, определяется следующим образом: P = I 2 x R = 6,024 2 x 16,6 = 602,3 Вт.
  • Расчет силы тока для каждого резистора выполняется по формулам: I 1 = U/R 1 = 100/200 = 0,5A; I 2 = U/R 2 = 100/100 = 1A; I 3 = U/R 3 = 100/51 = 1,96A; I 4 = U/R 4 = 100/39 = 2,56A. На примере этих сопротивлений прослеживается закономерность, что с уменьшением сопротивления, сила тока увеличивается.

Существует еще одна формула, позволяющая рассчитать мощность при паралл ельном подключении резисторов: P 1 = U 2 /R 1 = 100 2 /200 = 50 Вт; P 2 = U 2 /R 2 = 100 2 /100 = 100 Вт; P 3 = U 2 /R 3 = 100 2 /51 = 195,9 Вт; P 4 = U 2 /R 4 = 100 2 /39 = 256,4 Вт. Сложив мощности отдельных резисторов, получится их общая мощность: Р = Р 1 +Р 2 +Р 3 +Р 4 = 50+100+195,9+256,4 = 602,3 Вт.

Таким образом, мощность при последовательном и паралл ельном соединении резисторов определяется разными способами, с помощью которых можно получить максимально точные результаты.

Всем доброго времени суток. В прошлой статье я рассмотрел , применительно к электрическим цепям, содержащие источники энергии. Но в основе анализа и проектирования электронных схем вместе с законом Ома лежат также законы баланса , называемым первым законом Кирхгофа, и баланса напряжения на участках цепи, называемым вторым законом Кирхгофа, которые рассмотрим в данной статье. Но для начала выясним, как соединяются между собой приёмники энергии и какие при этом взаимоотношения между токами, напряжениями и .

Приемники электрической энергии можно соединить между собой тремя различными способами: последовательно, параллельно или смешано (последовательно — параллельно). Вначале рассмотрим последовательный способ соединения, при котором конец одного приемника соединяют с началом второго приемника, а конец второго приемника – с началом третьего и так далее. На рисунке ниже показано последовательное соединение приемников энергии с их подключением к источнику энергии

Пример последовательного подключения приемников энергии.

В данном случае цепь состоит из трёх последовательных приемников энергии с сопротивлением R1, R2, R3 подсоединенных к источнику энергии с U. Через цепь протекает электрический ток силой I, то есть, напряжение на каждом сопротивлении будет равняться произведению силы тока и сопротивления

Таким образом, падение напряжения на последовательно соединённых сопротивлениях пропорциональны величинам этих сопротивлений.

Из вышесказанного вытекает правило эквивалентного последовательного сопротивления, которое гласит, что последовательно соединённые сопротивления можно представить эквивалентным последовательным сопротивлением величина, которого равна сумме последовательно соединённых сопротивлений. Это зависимость представлена следующими соотношениями

где R – эквивалентное последовательное сопротивление.

Применение последовательного соединения

Основным назначением последовательного соединения приемников энергии является обеспечение требуемого напряжения меньше, чем напряжение источника энергии. Одними из таких применений является делитель напряжения и потенциометр


Делитель напряжения (слева) и потенциометр (справа).

В качестве делителей напряжения используют последовательно соединённые резисторы, в данном случае R1 и R2, которые делят напряжение источника энергии на две части U1 и U2. Напряжения U1 и U2 можно использовать для работы разных приемников энергии.

Довольно часто используют регулируемый делитель напряжения, в качестве которого применяют переменный резистор R. Суммарное сопротивление, которого делится на две части с помощью подвижного контакта, и таким образом можно плавно изменять напряжение U2 на приемнике энергии.

Ещё одним способом соединения приемников электрической энергии является параллельное соединение, которое характеризуется тем, что к одним и тем же узлам электрической цепи присоединены несколько преемников энергии. Пример такого соединения показан на рисунке ниже


Пример параллельного соединения приемников энергии.

Электрическая цепь на рисунке состоит из трёх параллельных ветвей с сопротивлениями нагрузки R1, R2 и R3. Цепь подключена к источнику энергии с напряжением U, через цепь протекает электрический ток с силой I. Таким образом, через каждую ветвь протекает ток равный отношению напряжения к сопротивлению каждой ветви

Так как все ветви цепи находятся под одним напряжением U, то токи приемников энергии обратно пропорциональны сопротивлениям этих приемников, а следовательно параллельно соединённые приемники энергии можно заметь одним приемником энергии с соответствующим эквивалентным сопротивлением, согласно следующих выражений

Таким образом, при параллельном соединении эквивалентное сопротивление всегда меньше самого малого из параллельно включенных сопротивлений.

Смешанное соединение приемников энергии

Наиболее широко распространено смешанное соединение приемников электрической энергии. Данной соединение представляет собой сочетание последовательно и параллельно соединенных элементов. Общей формулы для расчёта данного вида соединений не существует, поэтому в каждом отдельном случае необходимо выделять участки цепи, где присутствует только лишь один вид соединения приемников – последовательное или параллельное. Затем по формулам эквивалентных сопротивлений постепенно упрощать данные участи и в конечном итоге приводить их к простейшему виду с одним сопротивлением, при этом токи и напряжения вычислять по закону Ома. На рисунке ниже представлен пример смешанного соединения приемников энергии


Пример смешанного соединения приемников энергии.

В качестве примера рассчитаем токи и напряжения на всех участках цепи. Для начала определим эквивалентное сопротивление цепи. Выделим два участка с параллельным соединением приемников энергии. Это R1||R2 и R3||R4||R5. Тогда их эквивалентное сопротивление будет иметь вид

В результате получили цепь из двух последовательных приемников энергии R 12 R 345 эквивалентное сопротивление и ток, протекающий через них, составит

Тогда падение напряжения по участкам составит

Тогда токи, протекающие через каждый приемник энергии, составят

Как я уже упоминал, законы Кирхгофа вместе с законом Ома являются основными при анализе и расчётах электрических цепей. Закон Ома был подробно рассмотрен в двух предыдущих статьях, теперь настала очередь для законов Кирхгофа. Их всего два, первый описывает соотношения токов в электрических цепях, а второй – соотношение ЭДС и напряжениями в контуре. Начнём с первого.

Первый закон Кирхгофа гласит, что алгебраическая сумма токов в узле равна нулю. Описывается это следующим выражением

где ∑ — обозначает алгебраическую сумму.

Слово «алгебраическая» означает, что токи необходимо брать с учётом знака, то есть направления втекания. Таким образом, всем токам, которые втекают в узел, присваивается положительный знак, а которые вытекают из узла – соответственно отрицательный. Рисунок ниже иллюстрирует первый закон Кирхгофа


Изображение первого закона Кирхгофа.

На рисунке изображен узел, в который со стороны сопротивления R1 втекает ток, а со стороны сопротивлений R2, R3, R4 соответственно вытекает ток, тогда уравнение токов для данного участка цепи будет иметь вид

Первый закон Кирхгофа применяется не только к узлам, но и к любому контуру или части электрической цепи. Например, когда я говорил о параллельном соединении приемников энергии, где сумма токов через R1, R2 и R3 равна втекающему току I.

Как говорилось выше, второй закон Кирхгофа определяет соотношение между ЭДС и напряжениями в замкнутом контуре и звучит следующим образом: алгебраическая сумма ЭДС в любом контуре цепи равна алгебраической сумме падений напряжений на элементах этого контура. Второй закон Кирхгофа определяется следующим выражением

В качестве примера рассмотрим ниже следующую схему, содержащую некоторый контур


Схема, иллюстрирующая второй закон Кирхгофа.

Для начала необходимо определится с направлением обхода контура. В принципе можно выбрать как по ходу часовой стрелки, так и против хода часовой стрелки. Я выберу первый вариант, то есть элементы будут считаться в следующем порядке E1R1R2R3E2, таким образом, уравнение по второму закону Кирхгофа будет иметь следующий вид

Второй закон Кирхгофа применяется не только к цепям постоянного тока, но и к цепям переменного тока и к нелинейным цепям.
В следующей статье я рассмотрю основные способы расчёта сложных цепей с использованием закона Ома и законов Кирхгофа.

Теория это хорошо, но без практического применения это просто слова.

Последовательное и параллельное соединения источников тока

Решение:
Внутреннее сопротивление элементов

Сопротивление параллельно включенных резисторов

Общая э. д. с. элементов e0=2e Согласно закону Ома для полной цепи

15 Сопротивления резисторов R1 и R2 и э. д. с. ε1 и ε2 источников тока в схеме, изображенной на рис. 127, известны. При какой э.д.с. ε3 третьего источника ток через резистор R3 не течет?

Решение:
Выберем направления токов I1, I2 и I3 через резисторы R1, R2 и R3, указанные на рис. 363. Тогда I3=I1+I2. Разность потенциалов между точками а и b будет равна

Если

Исключая I1 находим

16 Цепь из трех одинаковых последовательно соединенных элементов с э.д.с. ε и внутренним сопротивлением r замкнута накоротко (рис. 128). Какое напряжение покажет вольтметр, подключенный к зажимам одного из элементов?

Решение:
Рассмотрим ту же схему без вольтметра (рис. 364). Из закона Ома для полной цепи находим

Из закона Ома для участка цепи между точками а и b получим

Подключение вольтметра к точкам, разность потенциалов между которыми равна нулю, ничего не может изменить в цепи. Поэтому вольтметр будет показывать напряжение, равное нулю.

17 Источник тока с э.д.с. ε0 включен в схему, параметры которой даны на рис. 129. Найти э.д.с. ε источника тока и направление его подключения к выводам а и b, при которых ток через резистор с сопротивлением R2 не идет.

Решение:
Подключим источник тока к выводам а и b и выберем направления токов, указанные на рис. 365. Для узла е имеем I=I0+I2. При обходе контуров aefb и ecdf по часовой стрелке получим

Используя условие I2 = 0, находим

Знак минус показывает, что полюсы источника тока на рис. 365 нужно поменять местами.

18 Два элемента с одинаковыми э.д.с. ε включены в цепь последовательно. Внешнее сопротивление цепи R = 5 Ом. Отношение напряжения на зажимах первого элемента к напряжению на зажимах второго элемента равно 2/3. Найти внутренние сопротивления элементов r1 и r2, если r1=2r2.

Решение:


19 Два одинаковых элемента с э.д.с. ε=1,5 В и внутренним сопротивлением r = 0,2 Ом замкнуты на резистор, сопротивление которого составляет в одном случае R1=0,2 Oм, В другом — R2 = 20 Ом. Как нужно соединить элементы (последовательно или параллельно) в первом и во втором случаях, чтобы получить наибольший ток в цепи?

Решение:
При параллельном соединении двух элементов внутреннее сопротивление и э.д.с. равны r/2 и ε при последовательном соединении они равны 2r и 2ε. Через резистор R при этом текут токи

Отсюда видно, что I2>I1, если R/2+r<R+r/2, т. е. если r1=r; следовательно, токи при параллельном и последовательном соединениях одинаковы. Во втором случае R2>r.Поэтому ток больше при последовательном соединении.

20 Два элемента с э.д.с. ε1=4В и ε2 = 2В и внутренними сопротивлениями r1 = 0,25 Ом и r2 = 0,75 Ом включены в схему, изображенную на рис. 130. Сопротивления резисторов R1 = 1 Ом и R2 = 3 Ом, емкость конденсатора С=2 мкФ. Найти заряд на конденсаторе.

Решение:


21 К батарее из двух параллельно включенных элементов с э.д.с. ε1 и ε2 и внутренними сопротивлениями r1 и r2 подключен резистор с сопротивлением R. Найти ток I, текущий через резистор R, и токи I1 и I2 в первом и втором элементах. При каких условиях токи в отдельных цепях могут быть равными нулю или изменять свое направление на обратное?

Решение:
Выберем направления токов, указанные на рис. 366. Для узла b имеем I-I1-I2=0. При обходе контуров abef и bcde по часовой стрелке получим

Из этих уравнений находим

Ток I=0 тогда, когда изменена полярность включения одного из элементов и, кроме того, выполнено условие

Ток I1=0 при

а ток I2 = 0 при

Токи I1 и I2 имеют направления, указанные на рис.366, если

Они меняют свое направление при

22 Батарея из n одинаковых аккумуляторов, соединенных в одном случае последовательно, в другом— параллельно, замыкается на резистор с сопротивлением R. При каких условиях ток, текущий через резистор, в обоих случаях будет один и тот же?

Решение:
При n(R-r) = R-r. Если R=r, то число элементов произвольно; если R№r, задача не имеет решения (n=1).

23 Батарея из n = 4 одинаковых элементов с внутренним сопротивлением r=2 Ом, соединенных в одном случае последовательно, в другом — параллельно, замыкается на резистор с сопротивлением R=10Ом. Во сколько раз показание вольтметра н одном случае отличается от показания вольтметра в другом случае? Сопротивление вольтметра велико по сравнению с R и r.

Решение:

где V1 — показание вольтметра при последовательном соединении элементов, V2-при параллельном.

24 Как изменится ток, текущий через резистор с сопротивлением R = 2 Ом, если n =10 одинаковых элементов, соединенных последовательно с этим резистором, включить параллельно ему? Э.д.с. элемента ε = 2 В, его внутреннее сопротивление r = 0,2 Ом.

Решение:


25 Батарея составлена из N=600 одинаковых элементов так, что n групп соединены последовательно и в каждой из них содержится т элементов, соединенных параллельно. Э.д.с. каждого элемента ε = 2 В, его внутреннее сопротивление r = 0,4 Ом. При каких значениях n и m батарея, будучи замкнута на внешнее сопротивление R = 0,6 Ом, отдаст во внешнюю цепь максимальную мощность? Найти при этом ток, текущий через сопротивление R.

Решение:
Общее число элементов N=nm (рис. 367). Ток во внешней цепи

где r/m— внутреннее сопротивление группы из т параллельно соединенных элементов, а nr/m — внутреннее сопротивление n групп, соединенных последовательно. Максимальная мощность отдается во внешнюю цепь при равенстве сопротивления R внутреннему сопротивлению батареи элементов nr/m, т. е.

При этом через сопротивление R течет точек I=46 А.

26 Емкость аккумулятора Qo=80А⋅ч. Найти емкость батареи из n = 3 таких аккумуляторов, включенных последовательно и параллельно.

Решение:
При последовательном соединении через все аккумуляторы батареи течет один и тот же ток, поэтому все они разрядятся в течение одного и того же времени. Следовательно, емкость батареи будет равна емкости каждого аккумулятора:
При параллельном соединении n аккумуляторов через каждый из них течет 1/n часть общего тока; поэтому при том же разрядном токе в общей цепи батареи будет разряжаться в n раз дольше, чем один аккумулятор, т. е. емкость батареи в п раз больше емкости отдельного аккумулятора:

Заметим, однако, что энергия

отдаваемая батареей в цепь, и при последовательном и при параллельном соединении n аккумуляторов в n раз больше энергии, отдаваемой одним аккумулятором. Это происходит потому, что при последовательном соединении э. д. с. батареи в n раз больше э. д. с. одного аккумулятора, а при параллельном соединении э.д.с. батареи остается той же, что и для каждого аккумулятора, но Q увеличивается в n раз.

27 Найти емкость батареи аккумуляторов, включенных по схеме, изображенной на рис.131. Емкость каждого аккумулятора Q0=64 А⋅ч.

Решение:
Каждая группа из пяти аккумуляторов, включенных последовательно, имеет емкость

Три параллельно включенные группы дают общую емкость батареи

28 Мост для измерения сопротивлений сбалансирован так, что ток через гальванометр не идет (рис. 132). Ток в правой ветви I=0,2 А. Найти напряжение V на зажимах источника тока. Сопротивления резисторов R1 = 2 Ом, R2 = 4 Ом, R3 = 1 Ом.

Решение:

29 Найти токи, протекающие в каждой ветви цепи, изображенной на рис. 133. Э.д.с. источников тока ε1 = 6,5 В и ε2 = 3,9 В. Сопротивления резисторов R1=R2=R3=R4=R5=R6=R=10 Ом.

Решение:
Составляем уравнения Кирхгофа в соответствии с направлениями токов, указанными на рис. 133: I1 + I2 — I3 = 0 для узла b;
I3 — I4 — I5 =0 для узла h; I5 — I1 — I6 = 0 для узла f: при этом

Для контура abfg (обход по часовой стрелке),

Для контура bcdh (обход против часовой стрелки) и

для контура hdef (обход по часовой стрелке). Решая эту систему уравнений с учетом, что все сопротивления одинаковы и равны R=10 Ом, получим

Отрицательные значения токов I2, I4 и I6 показывают, что при данных э.д.с. источников и сопротивлениях резисторов эти токи текут в стороны, противоположные указанным на рис. 133.

Соединение элементов питания и батарей

Источники напряжения обычно называют источниками питания. Для увеличения тока или напряжения, а может и того и другого источники питания (элементы, батареи) могут соединяться вместе. Существует три типа соединения элементов питания:
1. Последовательное соединение элементов.
2. Параллельное соединение элементов.
3. Последовательно-параллельное (смешанное) соединение элементов.

Последовательное соединение элементов.

При последовательном соединении элементов питания выделяются две схемы: последовательно-дополняющая и последовательно-препятствующая.
В последовательно-дополняющей схеме положительный вывод первого элемента питания соединяется с отрицательным выводом второго элемента питания; положительный вывод второго элемента питания соединяется с отрицательным выводом третьего элемента питания и т.д. (рисунок 3.11.)

Рисунок 3.11.Последовательное соединение элементов питания.

При таком соединении источников питания через все элементы будет течь одинаковый ток:

Iобщ=I1=I2=I3

Индексы в обозначениях токов указывают на номера отдельных источников питания (элементов или батарей питания)
А полное напряжение при последовательном соединении равно сумме напряжений (ЭДС) отдельных элементов:

Еобщ = Е1 + Е2 + Е3.

При последовательно-препятствующем включении источников питания, они соединяются друг с другом одноименными выводами. Но на практике такая схема не применяется или применяется, но очень редко.

Параллельное соединение элементов.

При параллельном соединении элементов питания, их одноименные выводы соединяются вместе, то есть плюс к плюсу, минус к минусу (рис 3.12).

Рисунок 3.11.Параллельное соединение элементов питания.

В этом случае общий ток будет равен сумме токов каждого элемента:

Iобщ=I1+I2+I3

Общее напряжение при параллельном включении источников питания будет равно напряжению каждого отдельного источника.

Еобщ = Е1 = Е2 = Е3.

Последовательно-параллельное соединение элементов напряжения.

Источники питания включают по последовательно-параллельной схеме для увеличения, как тока, так и напряжения. При этом основываются на том, что параллельное включение увеличивает силу тока, а последовательное увеличивает общее напряжение. На рисунке 3.13 показаны примеры последовательно-параллельных схем включения элементов питания.

Рисунок 3.11.Последовательно-параллельное соединение элементов питания.

ПОНРАВИЛАСЬ СТАТЬЯ? ПОДЕЛИСЬ С ДРУЗЬЯМИ В СОЦИАЛЬНЫХ СЕТЯХ!

Похожие материалы:

Добавить комментарий

Последовательное и параллельное соединение

В данной статье речь пойдет о последовательном и параллельном соединении проводников. На примерах будут рассмотрены данные соединения и как при таких соединениях будут изменяться такие величины как:

  • ток;
  • напряжение;
  • сопротивление.

В таблице 1.8 [Л2, с.24] приведены схемы и формулы по определению сопротивлений, токов и напряжений при параллельном и последовательном соединении.

Последовательное соединение

Последовательным соединением называются те участки цепи, по которым всегда проходят одинаковые токи.

При последовательном соединении:

  • сила тока во всех проводниках одинакова;
  • напряжение на всём соединении равно сумме напряжений на отдельных проводниках;
  • сопротивление всего соединения равно сумме сопротивлений отдельных проводников.

Пример 1

Последовательно подключены две лампы накаливания одинаковой мощности Рл1=Рл2=100 Вт к сети с напряжением Uн=220В. Сопротивление нити в лампах составляет Rл1=Rл2=122 Ом. Номинальное напряжение для ламп равно 220 B. На рис.1 показано последовательное включение ламп.

Решение

Составляем схему замещения, выражая каждую из входящих элементов цепи (в данном случае лампы накаливания) в виде сопротивлений.

1. Определяем ток протекающей по участкам цепи:

Iн = Uн/Rл1+ Rл2 = 220/122+122 = 0,9 A

2. Определяем напряжение на каждой из ламп накаливания, так как мощность ламп у нас одинаковая, то и напряжение для каждой из ламп будет одинаково:

Uл1=Uл2 = Iн*R = 0,9*122 = 110 B

Как мы видим напряжение источника (в данном примере 220 В) разделиться поровну, между обоими последовательно включенными лампами. При этом лампы будут ели светит, их накал будет неполным.

Для того чтобы лампы горели с полным накалом, нужно увеличить напряжение источника с 220В до 440В, при этом на каждой из ламп установиться номинальное (рабочее) напряжение равное 220В.

Пример 2

Последовательно подключены две лампы накаливания мощность Рл1 = 100 Вт и Рл2 = 75 Вт к сети с напряжением Uн=220В. Сопротивление нити в лампах составляют Rл1= 122 Ом для стоваттной лампы и Rл2= 153 Ом для семидесяти пяти ватной лампы.

Решение

1. Определяем ток протекающей по участкам цепи:

Iн = Uн/Rл1+ Rл2 = 220/100+75 = 0,8 A

2. Определяем напряжение на каждой из ламп накаливания:

Uл1= Iн*Rл1 = 0,8*122 = 98 B
Uл2= Iн*Rл2 = 0,8*153 = 122 B

Исходя из результатов расчетов, более мощная лампа 100 Вт получает при этом меньшее напряжение. Но ток в двух последовательно включенных даже разных лампах остается одинаковым. Например, если одна из ламп перегорит (порвется ее нить накаливания), погаснут обе лампы.

Данное соединение лампочек, например, используется в трамвайном вагоне для освещения салона.

Параллельное соединение

Параллельное соединение – это соединение, при котором начала всех проводников присоединяются к одной точке цепи, а их концы к другой.

Точки цепи, к которым сходится несколько проводов, называют узлами. Участки цепи, соединяющие между собой узлы, называют ветвями.

При параллельном соединении:

  • напряжение на всех проводниках одинаково;
  • сила тока в месте соединения проводников равна сумме токов в отдельных проводниках;
  • величина, обратная сопротивлению всего соединения, равна сумме величин, обратных сопротивлениям отдельных проводников.

Пример 3

Определить токи и напряжения всех участков цепи (рис.5), если известно:

  • Номинальное напряжение сети Uн = 220В;
  • Сопротивление нити в лампах HL1 и HL2 составляют Rл1 = Rл2 = 122 Ом.
  • Сопротивление нити в лампе HL3 составляют Rл3 = 153 Ом.

Решение

Составляем схему замещения для схемы, представленной на рис.5.

1. Определяем проводимость всей цепи [Л1, с.47] и согласно таблицы 1.8:

2. Определяем сопротивление всей цепи [Л1, с.47]:

3. Определяем силу тока цепи по закону Ома:

4. Определяем токи для каждой цепи [Л1, с.47]:

5. Выполним проверку, согласно которой, сила тока в месте соединения проводников равна сумме токов в отдельных проводниках:

Iл1+ Iл2+ Iл3=Iобщ.=1,8+1,8+1,44=5,04=5,04 (условие выполняется)

Смешанное соединение

Смешанным соединением – называется последовательно-параллельное соединение сопротивлений или участков цепи.

Пример 4

Определить токи и напряжения всех участков цепи (рис.7), если известно:

  • Номинальное напряжение сети Uн = 220В;
  • Сопротивление нити в лампах HL1, HL2, HL3 составляют Rл1 = Rл2 = Rл3 = 122 Ом.
  • Сопротивление нити в лампе HL4 составляют Rл4 = 153 Ом.
  • Результаты расчетов для участка цепи ВС (параллельное соединение проводников) применим из примера 3:
    Сопротивление цепи ВС составляет Rвс = 43,668 Ом.

Решение

Составляем схему замещения для схемы, представленной на рис.7.

1. Определяем сопротивление всей цепи:

Rобщ = Rав+Rвс = Rл1+Rвс = 122+43,688 = 165,688 Ом

2. Определяем силу тока цепи, согласно закона Ома:

3. Определяем напряжение на первом сопротивлении:

Uав=Uл1= Iобщ*Rл1 = 1,33*122 = 162 B

4. Определяем напряжение на участке ВС:

Uвс= Iобщ*Rвс = 1,33*43,688 = 58,1 B

5. Определяем токи для каждой цепи участка ВС:

6. Выполним проверку для участка цепи ВС:

Iл2+ Iл3+ Iл4= Iобщ.=0,48+0,48+0,38=1,33=1,33 (условие выполняется)

Литература:

  1. Общая электротехника с основами электроники, В.С. Попов, 1972 г.
  2. Справочная книга электрика. В.И. Григорьева. 2004 г.

Всего наилучшего! До новых встреч на сайте Raschet.info.

Как я могу рассчитать падение напряжения в параллельной цепи?

Что ж, для этого вам понадобится схема, чтобы показать мне, тем не менее, я даю вам два примера того, как работать.

См. Схему ниже,

Понятно, что Омега # 2 # параллельна другой Омега # 2 #, и их результирующее соединение последовательно с # 1 Ом # и # 3 Ом #, поэтому общее сопротивление цепи равно # (2 * 2) / (2 + 2) + 1 + 3 = 5 Ом #

Итак, ток, протекающий по цепи, равен # 20/5 = 4 А #

, поэтому падение потенциала на каждом сопротивлении можно рассчитать, просто умножив их значение сопротивления на ток, протекающий через них.

таким образом #V_ (1Ohm) = 1 * 4 = 4V #

#V_ (3 Ом) = 3 * 4 = 12 В #

и, #V_ (2 || 2 Ом) = (20-12-4) = 4 В #

или, вы могли бы умножить ток, протекающий через них, на их эквивалентное сопротивление, то есть # (2 * 2) / (2 + 2) * 4 = 1 * 4 = 4V #

В приведенном ниже примере в верхнем проводе # 1 Omega # параллельно # 1Omega #, их результат последовательно с # 0.5 Omega #, и эквивалентное сопротивление снова параллельно эквивалентному сопротивлению # 4 Omega. # и # 4 Омега # в параллельной комбинации.

Итак, чистое сопротивление верхнего провода составляет # (1 * 1) / (1 + 1) + 0,5 = 1 Омега #

.

, а нижний провод — # (4 * 4) / (4 + 4) = 2 Omega #

.

Итак, сопротивление цепи равно # (1 * 2) / (1 + 2) = 2/3 Омега #

.

Итак, ток, протекающий по цепи, равен # 14 / (2/3) = 21 А #

, падение напряжения на параллельной комбинации двух резисторов # 4 Omega # аналогично падению напряжения на батарее, то есть # 14 В #

на верхний провод упало такое же напряжение, поскольку оба соединены параллельно.

Итак, мы должны найти ток, протекающий через верхнюю цепь, чтобы вычислить это.

Итак, ток, протекающий по нижнему проводу, равен # 14/2 = 7A # (поскольку чистое сопротивление нижнего провода составляет # 2 Омега #

, таким образом, ток, протекающий через верхнюю цепь, равен # (21-7) = 14 A #

Итак, падение потенциала на # 0,5 Омега # составляет # 14 * 0,5 = 7 В #

Итак, остальные, то есть # (14-7) = 7 V #, должны были упасть на параллельную комбинацию резисторов # 1 Omega #.

Простые параллельные схемы | Последовательные и параллельные схемы

На этой странице мы изложим три принципа, которые вы должны понимать в отношении параллельных цепей:

  1. Напряжение: Напряжение одинаково на всех компонентах в параллельной цепи.
  2. Ток: Полный ток цепи равен сумме токов отдельных ответвлений.
  3. Сопротивление: Отдельные сопротивления уменьшают , чтобы равняться меньшему общему сопротивлению, вместо прибавляют , чтобы получить общее.

Давайте взглянем на несколько примеров параллельных цепей, демонстрирующих эти принципы.

Начнем с параллельной схемы, состоящей из трех резисторов и одной батареи:

Напряжение в параллельных цепях

Первый принцип для понимания параллельных цепей заключается в том, что напряжение одинаково на всех компонентах в цепи .Это связано с тем, что в параллельной цепи есть только два набора электрически общих точек, и напряжение, измеренное между наборами общих точек, всегда должно быть одинаковым в любой момент времени.

Следовательно, в приведенной выше схеме напряжение на R 1 равно напряжению на R 2 , которое равно напряжению на R 3 , которое равно напряжению на батарее.

Это равенство напряжений можно представить в другой таблице для наших начальных значений:

Применение закона Ома для простых параллельных цепей

Как и в случае с последовательными цепями, применимо то же предостережение для закона Ома: значения напряжения, тока и сопротивления должны быть в одном контексте, чтобы вычисления работали правильно.

Однако в приведенной выше примерной схеме мы можем немедленно применить закон Ома к каждому резистору, чтобы найти его ток, потому что мы знаем напряжение на каждом резисторе (9 вольт) и сопротивление каждого резистора:

На данный момент мы все еще не знаем, каков полный ток или полное сопротивление для этой параллельной цепи, поэтому мы не можем применить закон Ома к крайнему правому столбцу («Всего»). Однако, если мы внимательно подумаем о том, что происходит, должно стать очевидным, что общий ток должен равняться сумме всех токов отдельных резисторов («ответвлений»):

По мере того, как полный ток выходит из положительной (+) клеммы аккумулятора в точке 1 и проходит через цепь, часть потока разделяется в точке 2, чтобы пройти через R 1 , еще часть разделяется в точке 3, чтобы пройти через R 2 , а оставшаяся часть идет через R 3 .Подобно реке, разветвляющейся на несколько меньших ручьев, общий расход всех потоков должен равняться расходу всей реки.

То же самое происходит, когда токи через R 1 , R 2 и R 3 соединяются, чтобы течь обратно к отрицательной клемме батареи (-) к точке 8: поток тока из точки 7 до точки 8 должно равняться сумме токов (ответвлений) через R 1 , R 2 и R 3 .

Это второй принцип параллельных цепей: полный ток цепи равен сумме токов отдельных ветвей .

Используя этот принцип, мы можем заполнить место ИТ на нашем столе суммой I R1 , I R2 и I R3 :

Как рассчитать полное сопротивление в параллельных цепях

Наконец, применив закон Ома к крайнему правому столбцу («Общее»), мы можем вычислить полное сопротивление цепи:

Уравнение сопротивления в параллельных цепях

Обратите внимание на кое-что очень важное. Общее сопротивление цепи составляет всего 625 Ом: на меньше , чем у любого из отдельных резисторов.В последовательной цепи, где полное сопротивление было суммой отдельных сопротивлений, общее сопротивление должно было быть на больше , чем у любого из резисторов по отдельности.

Здесь, в параллельной цепи, наоборот: мы говорим, что отдельные сопротивления уменьшают , а не добавляют , чтобы получить общее .

Этот принцип завершает нашу триаду «правил» для параллельных цепей, точно так же, как было обнаружено, что у последовательных цепей есть три правила для напряжения, тока и сопротивления.

Математически соотношение между общим сопротивлением и отдельными сопротивлениями в параллельной цепи выглядит следующим образом:

Как изменить схемы нумерации параллельных цепей для SPICE

Та же основная форма уравнения работает для любого числа резисторов, соединенных вместе параллельно, просто добавьте столько членов 1 / R к знаменателю дроби, сколько необходимо для размещения всех параллельных резисторов в цепи.

Как и в случае с последовательной схемой, мы можем использовать компьютерный анализ для перепроверки наших расчетов.Во-первых, конечно, мы должны описать нашу примерную схему компьютеру в понятных ему терминах. Я начну с рисования схемы:

И снова мы обнаруживаем, что исходная схема нумерации, используемая для идентификации точек в цепи, должна быть изменена в интересах SPICE.

В SPICE все электрически общие точки должны иметь одинаковые номера узлов. Так SPICE узнает, что с чем связано и как.

В простой параллельной схеме все точки электрически являются общими в одном из двух наборов точек.В нашей примерной схеме провод, соединяющий верхние части всех компонентов, будет иметь один номер узла, а провод, соединяющий низ компонентов, будет иметь другой номер.

Оставаясь верным соглашению о включении нуля в качестве номера узла, я выбираю числа 0 и 1:

Пример, подобный этому, делает обоснование номеров узлов в SPICE довольно ясным для понимания. Поскольку все компоненты имеют общие наборы чисел, компьютер «знает», что все они подключены параллельно друг другу.

Чтобы отобразить токи ответвлений в SPICE, нам нужно вставить источники нулевого напряжения последовательно (последовательно) с каждым резистором, а затем привязать наши измерения тока к этим источникам.

По какой-то причине создатели программы SPICE сделали так, чтобы ток можно было рассчитать только с по от источника напряжения. Это несколько раздражающее требование программы моделирования SPICE. После добавления каждого из этих «фиктивных» источников напряжения необходимо создать несколько новых номеров узлов, чтобы подключить их к соответствующим резисторам ответвления:

Как проверить результаты компьютерного анализа

Все фиктивные источники напряжения настроены на 0 вольт, чтобы не влиять на работу схемы.

Файл описания схемы или список цепей выглядит так:

Параллельная схема
 v1 1 0
 r1 2 0 10к
 r2 3 0 2k
 r3 4 0 1k
 vr1 1 2 постоянного тока 0
 vr2 1 3 постоянного тока 0
 vr3 1 4 постоянного тока 0
 .dc v1 9 9 1
 .print dc v (2,0) v (3,0) v (4,0)
 .print dc i (vr1) i (vr2) i (vr3)
 .конец
 

Запустив компьютерный анализ, мы получили следующие результаты (я снабдил распечатку описательными этикетками):

версия 1 в (2) в (3) в (4)
9.000E + 00 9.000E + 00 9.000E + 00 9.000E + 00
аккумулятор Напряжение R1 R2 напряжение R3 напряжение

Напряжение

версия 1 я (vr1) i (vr2) я (vr3)
9.000E + 00 9.000E-04 4.500E-03 9.000E-03
аккумулятор R1 ток R2 текущий R3 ток

Напряжение

Эти значения действительно совпадают со значениями, вычисленными ранее по закону Ома: 0.9 мА для I R1 , 4,5 мА для I R2 и 9 мА для I R3 . При параллельном подключении, естественно, все резисторы имеют одинаковое падение напряжения на них (9 вольт, как у батареи).

Три правила параллельных цепей

Таким образом, параллельная цепь определяется как цепь, в которой все компоненты подключены между одним и тем же набором электрически общих точек. Другими словами, все компоненты подключены друг к другу через клеммы.Из этого определения следуют три правила параллельных цепей:

  • Все компоненты имеют одинаковое напряжение.
  • Сопротивления уменьшаются до меньшего общего сопротивления.
  • Токи ответвления в сумме равняются большему общему току.

Как и в случае с последовательными цепями, все эти правила находят корень в определении параллельной цепи. Если вы полностью понимаете это определение, то правила — не что иное, как сноски к определению.

ОБЗОР:

  • Компоненты в параллельной цепи имеют одинаковое напряжение: E Всего = E 1 = E 2 =. . . E n
  • Общее сопротивление в параллельной цепи на меньше , чем любое из отдельных сопротивлений: R Всего = 1 / (1 / R 1 + 1 / R 2 +… 1 / R n )
  • Общий ток в параллельной цепи равен сумме токов отдельных ответвлений: I Всего = I 1 + I 2 +.. . Я .

СВЯЗАННЫЕ РАБОЧИЕ ЛИСТЫ:

Как я могу рассчитать падение напряжения в параллельном контуре, класс 12, физика CBSE

Подсказка: Это можно решить, используя закон Ома, который гласит, что напряжение на проводнике прямо пропорционально току, протекающему по нему, а для параллельных цепей мы будет использовать формулу для общего сопротивления, чтобы найти полное падение напряжения.

Используемая формула:
$ \ begin {align}
& V = IR \\
& R = {{R} _ {1}} + {{R} _ {2}} + {{R} _ { 3}} \\
\ end {align} $

Полное пошаговое решение:
Падение напряжения на резисторе в параллельной цепи одинаково для всех резисторов в каждой ветви всех параллельных цепей в схема параллельной цепи.

Напряжение, выраженное в напряжении, измеряет электродвижущие силы, управляющие цепью, или разность потенциалов.

Рассмотрим схему с тремя резисторами, включенными параллельно, с источником напряжения «V» и током, протекающим в «I».


Теперь пошаговый метод расчета напряжения в цепи.
Шаг 1: — Найдите полное сопротивление параллельной цепи.
Формула для полного сопротивления цепи, соединенной параллельно,
$ \ begin {align}
& \ Rightarrow \ dfrac {1} {{{R} _ {T}}} = \ dfrac {1} {{R} _ {1}}} + \ dfrac {1} {{{R} _ {2}}} + \ dfrac {1} {{{R} _ {3}}} \\
и \ следовательно {{R} _ {T}} = \ dfrac {{{R} _ {1}} {{R} _ {2}} {{R} _ {3}}} {{{R} _ {1}} {{R } _ {2}} + {{R} _ {2}} {{R} _ {3}} + {{R} _ {1}} {{R} _ {3}}}…. \ left (3 \ right) \\
\ end {align} $

Теперь по закону Ома
$ \ Rightarrow V = I {{R} _ {T}} $
Где, V = напряжение
I = ток
$ {{R} _ {T}} $ = общее сопротивление
$ \ Rightarrow V = \ dfrac {I \ times {{R} _ {1}} {{R} _ {2}} { {R} _ {3}}} {{{R} _ {1}} {{R} _ {2}} + {{R} _ {2}} {{R} _ {3}} + {{ R} _ {3}} {{R} _ {4}}} …. \ left (1 \ right) $
Рассмотрите этот метод, потому что согласно действующему закону Кирхгофа, величина тока, входящего в цепь, равна равна величине тока, выходящего из цепи.

Примечание:
В качестве подхода к решению этого вопроса мы используем закон Ома, чтобы найти напряжение на цепи резистора, которые подключены параллельно, используя общее сопротивление параллельной цепи и общее количество тока, протекающего в цепи.

Как рассчитать падение напряжения на резисторе, подробное объяснение

Если вы ищете, как рассчитать падение напряжения на резисторе, то SoManyTech предлагает вам полную теорию и практические примеры падения напряжения на резисторе.Перед этим давайте освежим в памяти закон Ома: (Прокрутите вниз, если вы профессиональный пользователь)

  • Распространенный способ показать поведение схемного устройства — это его характеристика.
    Это график зависимости тока «I» через устройство от приложенного к нему напряжения «V». Это устройство, резистор, имеет простую линейную характеристику V – I , показанную на рис. выше.
  • Эта линейная зависимость устройства выражается законом Ома :
    V = IR
  • Константа пропорциональности R известна как сопротивление устройства и равна наклону кривой I. –V характеристика.Единица измерения сопротивления — Ом, обозначение — Ом . Любое устройство с линейной ВАХ называется резистором.

Какое падение напряжения на резисторе?

  • Падение напряжения на резисторе — это не что иное, как значение напряжения на резисторе. Иногда его также называют «напряжение на резисторе» или просто «падение напряжения».
  • Обычно обозначается как:
    ‘V (drop ) ‘ или ‘Vr’ или ‘Vd’
    Для нескольких резисторов это записывается как Vr1, Vr2, Vr3 и т. Д.

Как мы все знаем, резистор — это устройство, которое оказывает сопротивление протекающему через него току. Затем, применяя закон Ома, резистор будет предлагать падение напряжения на резистивном устройстве, которое определяется как:

В (падение ) = I × R

где, I = ток через резистор в (А) амперах
R = сопротивление в (Ом) Ом
В (падение ) = падение напряжения в (В) вольтах

Как рассчитать падение напряжения на сопротивлении по шагам:

Шаг 1: Упростим данную схему.Если, скажем, схема заполнена резисторами, включенными последовательно и параллельно, то повторно подключите ее, чтобы упростить. (проверьте практический пример ниже)

Step2: Затем найдите эквивалентный резистор.
Для параллельного подключения: 1 / Треб. = 1 / R1 + 1 / R2…
Для серии: Треб. = R1 + R2 +. . .

Step3: Найдите ток через каждый резистор. (Ток через последовательный резистор такой же, а ток через параллельные резисторы отличается и зависит от его значения)

Step4 : Примените формулу закона Ома для расчета падения напряжения.
В = IR

Напряжение в последовательной цепи — Практические примеры:

Случай I:

Если есть только один резистор последовательно с батареей или источником питания, как показано в этой схеме.


В этой схеме падение напряжения на резисторе такое же, как и в цепи питания. Это связано с тем, что оба компонента имеют общие потенциальные точки, разделенные между ними (точка A и точка B)

Vs = Vdrop = 5 вольт (скажем)

Случай II:

Если есть два или несколько резисторов, включенных последовательно с батареей, как показано на этой схеме.

В этой схеме мы должны вычислить полный ток «I», протекающий через цепь.
I (общий) = V (питание) / R (эквивалент)

∴ I (общий) = 5/30 = 0,166 A

Тогда падение напряжения на R1 будет: Vr1 = I × R1
Падение напряжения на R2 будет: Vr2 = I × R2
Падение напряжения на Rn будет: Vrn = I × Rn

  • Vr1 = I × R1 = 0,166 × 10 = 1,66 В & Vr2 = I × R2 = 0.166 × 20 = 3,33 В

Напряжение на параллельных резисторах:

Случай I:
Два резистора включены параллельно батарее или источнику питания, как показано на этой схеме.


В этой схеме падение напряжения на этих параллельных резисторах такое же, как у источника питания.
Это связано с тем, что оба резистора имеют общие потенциальные точки, разделенные между ними (точка A и точка B), поэтому напряжение будет одинаковым, но ток будет другим.

∴Vs = Vdrop = Vr1 = Vr2 = 5 вольт (скажем)

Случай II:
Один резистор включен последовательно и два резистора с источником питания, как показано на этой схеме.

В этой схеме нам нужно вычислить ток «I» через каждый компонент.

  • i1 = I (всего) = Is = V (питание) / R (эквивалент)

    где, R (эквивалент) = R1 + Rp
    , где 1 / Rp = 1 / R2 + 1 / R3

    ∴ Rp = 12 Ом и R ( эквивалент ) = 22 Ом

  • i2 = i1 * (R3 / (R2 + R3))
    i3 = i1 * (R2 / (R2 + R3))
  • Падение напряжения на R1 будет Vr1 = R1 * i1
    Падение напряжения на R2 будет Vr2 = R2 * i2
    Падение напряжения на R3 будет Vr3 = R2 * i3

Установка значения, которые мы имеем,

Теперь i1 = V (поставка) / R (эквивалент) = 5/22 = 0.227 ампер
i1 = 0,227 A

Падение напряжения на 10 Ом -> Vr1 = 10 * i1 = 10 × 0,227 вольт
Vr1 = 2,27 вольт

Теперь, i2 = i1 * (R3 / (R1 + R2))
i2 = 0,1362 A

Падение напряжения на 20 Ом -> Vr2 = 20 * i2 = 20 × 0,1362 В
Vr2 = 2,724 В

Теперь i3 = i1 * (R2 / (R1 + R2))
i3 = 0.09 A

Падение напряжения на 30 Ом -> Vr2 = 30 * i2 = 30 × 0,09 В
∴ Vr3 = 2,7 В

Метод 2:

  • Найти i1 = В (питание) / R (эквивалент) = 0,227 A
    Тогда падение напряжения на R1 будет Vr1 = R1 * i1 = 10 × 0,227 = 2,27 В

    ∴ Эквивалентное напряжение в точке ‘A’ будет равно
    Veq = Va = Vs — Vr1

    ∴ Va = 5-2.27 = 2,73 В
    Следовательно, мы получаем одинаковое значение потенциала на R2 и R3.

  • Таким образом, Va = Vr2 = Vr3 = 2,73 вольт

Метод 3:

В этом методе мы используем цифровой мультиметр или, можно сказать, вольтметр. Все, что вам нужно, это установить мультиметр в режим напряжения.
Теперь с помощью двух щупов проверьте напряжение на требуемом резисторе, подключив к нему щупы. (на рис. показания вольтметра только для справки)

Вуаля !! Ты понял.

Это самый простой способ найти падение напряжения на резисторе в любой цепи.

Последовательные и параллельные резисторы

Цели обучения

К концу этого раздела вы сможете:

  • Нарисуйте цепь с резисторами, включенными параллельно и последовательно.
  • Рассчитайте падение напряжения на резисторе по закону Ома.
  • Контраст — способ расчета общего сопротивления для резисторов, включенных последовательно и параллельно.
  • Объясните, почему полное сопротивление параллельной цепи меньше наименьшего сопротивления любого из резисторов в этой цепи.
  • Рассчитайте общее сопротивление цепи, которая содержит смесь резисторов, включенных последовательно и параллельно.

Большинство схем имеет более одного компонента, называемого резистором , который ограничивает поток заряда в цепи. Мера этого предела расхода заряда называется сопротивлением . Простейшие комбинации резисторов — это последовательное и параллельное соединение, показанное на Рисунке 1.Общее сопротивление комбинации резисторов зависит как от их индивидуальных значений, так и от способа их подключения.

Рис. 1. (a) Последовательное соединение резисторов. (б) Параллельное соединение резисторов.

Когда резисторы в серии ? Резисторы включены последовательно всякий раз, когда поток заряда, называемый током , должен проходить через устройства последовательно. Например, если ток протекает через человека, держащего отвертку, и попадает в Землю, тогда R 1 на Рисунке 1 (а) может быть сопротивлением вала отвертки, R 2 сопротивлением ее ручки , R 3 сопротивление тела человека и R 4 сопротивление его обуви.На рисунке 2 показаны резисторы, последовательно подключенные к источнику напряжения . Кажется разумным, что полное сопротивление является суммой отдельных сопротивлений, учитывая, что ток должен проходить через каждый резистор последовательно. (Этот факт был бы преимуществом для человека, желающего избежать поражения электрическим током, который мог бы уменьшить ток, надев обувь с высоким сопротивлением на резиновой подошве. прибор, уменьшающий рабочий ток.)

Рис. 2. Три резистора, подключенных последовательно к батарее (слева), и эквивалентное одиночное или последовательное сопротивление (справа).

Чтобы убедиться, что последовательно включенные сопротивления действительно складываются, давайте рассмотрим потерю электроэнергии, называемую падением напряжения , в каждом резисторе на рисунке 2. Согласно закону Ома , падение напряжения В на резистор, когда через него протекает ток, рассчитывается по формуле V = IR , где I равно току в амперах (A), а R — сопротивление в омах (Ω).Другой способ представить это: В — это напряжение, необходимое для протекания тока I через сопротивление R . Таким образом, падение напряжения на R 1 составляет В 1 = IR 1 , что на R 2 составляет В 2 = IR 2 и что для R 3 это V 3 = IR 3 .Сумма этих напряжений равна выходному напряжению источника; то есть

В = В 1 + В 2 + В 3 .

Это уравнение основано на сохранении энергии и сохранении заряда. Электрическая потенциальная энергия может быть описана уравнением: PE = qV , где q — электрический заряд, а В, — напряжение. Таким образом, энергия, отдаваемая источником, составляет кв , а рассеиваемая резисторами —

.

qV 1 + qV 2 + qV 3 .

Установление связей: законы о сохранении

Вывод выражений для последовательного и параллельного сопротивления основан на законах сохранения энергии и сохранения заряда, которые утверждают, что общий заряд и полная энергия постоянны в любом процессе. Эти два закона непосредственно участвуют во всех электрических явлениях и будут многократно использоваться для объяснения как конкретных эффектов, так и общего поведения электричества.

Эти энергии должны быть равны, потому что в цепи нет другого источника и другого назначения для энергии.Таким образом, qV = qV 1 + qV 2 + qV 3 . Сбор q аннулируется, давая V = V 1 + V 2 + V 3 , как указано. (Обратите внимание, что одинаковое количество заряда проходит через батарею и каждый резистор за заданный промежуток времени, поскольку нет емкости для хранения заряда, нет места для утечки заряда и заряд сохраняется.) Теперь подстановка значений для отдельных напряжений дает

В = IR 1 + IR 2 + IR 3 = I ( R 1 + R 2 + R 3 ).

Обратите внимание, что для эквивалентного сопротивления одной серии R s , мы имеем

В = ИК с .

Это означает, что полное или эквивалентное последовательное сопротивление R s трех резисторов составляет R s = R 1 + R 2 + R 3 .Эта логика действительна в общем для любого количества резисторов, включенных последовательно; таким образом, полное сопротивление R s последовательного соединения составляет

R с = R 1 + R 2 + R 3 +…,

, как предлагается. Поскольку весь ток должен проходить через каждый резистор, он испытывает сопротивление каждого, а последовательно соединенные сопротивления просто складываются.

Пример 1. Расчет сопротивления, тока, падения напряжения и рассеиваемой мощности: анализ последовательной цепи

Предположим, что выходное напряжение батареи на рисунке 2 равно 12.0 В, а сопротивления равны R 1 = 1,00 Ом, R 2 = 6,00 Ом и R 3 = 13,0 Ом. а) Каково полное сопротивление? (б) Найдите ток. (c) Рассчитайте падение напряжения на каждом резисторе и покажите, как они складываются, чтобы равняться выходному напряжению источника. (d) Рассчитайте мощность, рассеиваемую каждым резистором. (e) Найдите выходную мощность источника и покажите, что она равна общей мощности, рассеиваемой резисторами.

Стратегия и решение для (а)

Общее сопротивление — это просто сумма отдельных сопротивлений, определяемая следующим уравнением:

[латекс] \ begin {array} {lll} {R} _ {\ text {s}} & = & {R} _ {1} + {R} _ {2} + {R} _ {3} \ \ & = & 1.00 \ text {} \ Omega + 6.00 \ text {} \ Omega + 13.0 \ text {} \ Omega \\ & = & 20.0 \ text {} \ Omega \ end {array} \\ [/ latex].

Стратегия и решение для (b)

Ток определяется по закону Ома: В = IR . Ввод значения приложенного напряжения и общего сопротивления дает ток для цепи:

[латекс] I = \ frac {V} {{R} _ {\ text {s}}} = \ frac {12.0 \ text {V}} {20.0 \ text {} \ Omega} = 0.60 \ text {A }\\[/латекс].

Стратегия и решение для (c)

Напряжение — или падение IR — на резисторе определяется законом Ома.Ввод значения тока и значения первого сопротивления дает

.

В 1 = IR 1 = (0,600 A) (1,0 Ом) = 0,600 В.

Аналогично

В 2 = IR 2 = (0,600 A) (6,0 Ом) = 3,60 В

и

V3 = IR 3 = (0,600 A) (13,0 Ом) = 7,80 В.

Обсуждение для (c)

Три капли IR добавляют к 12.0 В, прогноз:

В 1 + В 2 + В 3 = (0,600 + 3,60 + 7,80) В = 12,0 В.

Стратегия и решение для (d)

Самый простой способ рассчитать мощность в ваттах (Вт), рассеиваемую резистором в цепи постоянного тока, — это использовать закон Джоуля , P = IV , где P — электрическая мощность. В этом случае через каждый резистор протекает одинаковый полный ток.Подставляя закон Ома В = IR в закон Джоуля, мы получаем мощность, рассеиваемую первым резистором, как

.

P 1 = I 2 R 1 = (0,600 A) 2 (1,00 Ом) = 0,360 Вт

Аналогично

P 2 = I 2 R 2 = (0,600 A) 2 (6,00 Ом) = 2,16 Вт.

и

P 3 = I 2 R 3 = (0.{2}} {R} \\ [/ latex], где В, — это падение напряжения на резисторе (а не полное напряжение источника). Будут получены те же значения.

Стратегия и решение для (e)

Самый простой способ рассчитать выходную мощность источника — использовать P = IV , где В, — напряжение источника. Это дает

P = (0,600 A) (12,0 В) = 7,20 Вт.

Обсуждение для (e)

Обратите внимание, что по совпадению общая мощность, рассеиваемая резисторами, также равна 7.20 Вт, столько же, сколько мощность, выдаваемая источником. То есть

P 1 + P 2 + P 3 = (0,360 + 2,16 + 4,68) W = 7,20 Вт.

Мощность — это энергия в единицу времени (ватт), поэтому для сохранения энергии требуется, чтобы выходная мощность источника была равна общей мощности, рассеиваемой резисторами.

Основные характеристики резисторов серии

  1. Последовательные сопротивления добавить: R с = R 1 + R 2 + R 3 +….
  2. Одинаковый ток течет через каждый резистор последовательно.
  3. Отдельные резисторы, включенные последовательно, не получают полное напряжение источника, а делят его.

На рисунке 3 показаны резисторы , подключенные параллельно , подключенные к источнику напряжения. Резисторы включены параллельно, когда каждый резистор подключен непосредственно к источнику напряжения с помощью соединительных проводов с незначительным сопротивлением. Таким образом, к каждому резистору приложено полное напряжение источника. Каждый резистор потребляет такой же ток, как если бы он один был подключен к источнику напряжения (при условии, что источник напряжения не перегружен).Например, автомобильные фары, радио и т. Д. Подключены параллельно, так что они используют полное напряжение источника и могут работать полностью независимо. То же самое и в вашем доме, или в любом другом здании. (См. Рисунок 3 (b).)

Рис. 3. (a) Три резистора, подключенных параллельно батарее, и эквивалентное одиночное или параллельное сопротивление. (б) Электроснабжение в доме. (Источник: Dmitry G, Wikimedia Commons)

Чтобы найти выражение для эквивалентного параллельного сопротивления R p , давайте рассмотрим протекающие токи и их связь с сопротивлением.Поскольку каждый резистор в цепи имеет полное напряжение, токи, протекающие через отдельные резисторы, равны [латекс] {I} _ {1} = \ frac {V} {{R} _ {1}} \\ [/ latex] , [латекс] {I} _ {2} = \ frac {V} {{R} _ {2}} \\ [/ latex] и [латекс] {I} _ {3} = \ frac {V} {{R} _ {3}} \\ [/ латекс]. Сохранение заряда подразумевает, что полный ток I , производимый источником, является суммой этих токов:

I = I 1 + I 2 + I 3 .

Подстановка выражений для отдельных токов дает

[латекс] I = \ frac {V} {{R} _ {1}} + \ frac {V} {{R} _ {2}} + \ frac {V} {{R} _ {3}} = V \ left (\ frac {1} {{R} _ {1}} + \ frac {1} {{R} _ {2}} + \ frac {1} {{R} _ {3}} \ справа) \\ [/ латекс].

Обратите внимание, что закон Ома для эквивалентного одиночного сопротивления дает

[латекс] I = \ frac {V} {{R} _ {p}} = V \ left (\ frac {1} {{R} _ {p}} \ right) \\ [/ latex].

Члены в скобках в последних двух уравнениях должны быть равны. Обобщая для любого количества резисторов, общее сопротивление R p параллельного соединения связано с отдельными сопротивлениями на

.

[латекс] \ frac {1} {{R} _ {p}} = \ frac {1} {{R} _ {1}} + \ frac {1} {{R} _ {2}} + \ гидроразрыв {1} {{R} _ {\ text {.} 3}} + \ text {.} \ Text {…} \\ [/ latex]

Это соотношение приводит к общему сопротивлению R p , которое меньше наименьшего из отдельных сопротивлений. (Это видно в следующем примере.) При параллельном подключении резисторов от источника течет больше тока, чем протекает по любому из них по отдельности, поэтому общее сопротивление ниже.

Пример 2. Расчет сопротивления, тока, рассеиваемой мощности и выходной мощности: анализ параллельной цепи

Пусть выходное напряжение батареи и сопротивления в параллельном соединении на Рисунке 3 будут такими же, как и в ранее рассмотренном последовательном соединении: В = 12.0 В, R 1 = 1,00 Ом, R 2 = 6,00 Ом и R 3 = 13,0 Ом. а) Каково полное сопротивление? (б) Найдите полный ток. (c) Рассчитайте токи в каждом резисторе и покажите, как они складываются, чтобы равняться общему выходному току источника. (d) Рассчитайте мощность, рассеиваемую каждым резистором. (e) Найдите выходную мощность источника и покажите, что она равна общей мощности, рассеиваемой резисторами.

Стратегия и решение для (а)

Общее сопротивление для параллельной комбинации резисторов находится с помощью следующего уравнения.Ввод известных значений дает

[латекс] \ frac {1} {{R} _ {p}} = \ frac {1} {{R} _ {1}} + \ frac {1} {{R} _ {2}} + \ frac {1} {{R} _ {3}} = \ frac {1} {1 \ text {.} \ text {00} \ text {} \ Omega} + \ frac {1} {6 \ text {. } \ text {00} \ text {} \ Omega} + \ frac {1} {\ text {13} \ text {.} 0 \ text {} \ Omega} \\ [/ latex].

Таким образом,

[латекс] \ frac {1} {{R} _ {p}} = \ frac {1.00} {\ text {} \ Omega} + \ frac {0 \ text {.} \ Text {1667}} {\ текст {} \ Omega} + \ frac {0 \ text {.} \ text {07692}} {\ text {} \ Omega} = \ frac {1 \ text {.} \ text {2436}} {\ text { } \ Omega} \\ [/ латекс].

(Обратите внимание, что в этих расчетах каждый промежуточный ответ отображается с дополнительной цифрой.Мы должны перевернуть это, чтобы найти полное сопротивление R p . Это дает

[латекс] {R} _ {\ text {p}} = \ frac {1} {1 \ text {.} \ Text {2436}} \ text {} \ Omega = 0 \ text {.} \ Text { 8041} \ text {} \ Omega \\ [/ latex].

Суммарное сопротивление с правильным количеством значащих цифр составляет R p = 0,804 Ом

Обсуждение для (а)

R p , как и предполагалось, меньше наименьшего индивидуального сопротивления.

Стратегия и решение для (b)

Полный ток можно найти из закона Ома, заменив полное сопротивление R p . Это дает

[латекс] I = \ frac {V} {{R} _ {\ text {p}}} = \ frac {\ text {12.0 V}} {0.8041 \ text {} \ Omega} = \ text {14} \ text {.} \ text {92 A} \\ [/ latex].

Обсуждение для (б)

Ток I для каждого устройства намного больше, чем для тех же устройств, подключенных последовательно (см. Предыдущий пример).Схема с параллельным соединением имеет меньшее общее сопротивление, чем резисторы, включенные последовательно.

Стратегия и решение для (c)

Отдельные токи легко вычислить по закону Ома, поскольку каждый резистор получает полное напряжение. Таким образом,

[латекс] {I} _ {1} = \ frac {V} {{R} _ {1}} = \ frac {12.0 \ text {V}} {1.00 \ text {} \ Omega} = 12.0 \ text {A} \\ [/ латекс].

Аналогично

[латекс] {I} _ {2} = \ frac {V} {{R} _ {2}} = \ frac {12.0 \ text {V}} {6.00 \ text {} \ Omega} = 2 \ text {.} \ text {00} \ text {A} \\ [/ latex]

и

[латекс] {I} _ {3} = \ frac {V} {{R} _ {3}} = \ frac {\ text {12} \ text {.} 0 \ text {V}} {\ text {13} \ text {.} \ Text {0} \ text {} \ Omega} = 0 \ text {.} \ Text {92} \ text {A} \\ [/ latex].

Обсуждение для (c)

Общий ток складывается из отдельных токов:

I 1 + I 2 + I 3 = 14,92 A.

Это соответствует сохранению заряда.{2}} {13.0 \ text {} \ Omega} = 11.1 \ text {W} \\ [/ latex].

Обсуждение для (d)

Мощность, рассеиваемая каждым резистором параллельно, значительно выше, чем при последовательном подключении к тому же источнику напряжения.

Стратегия и решение для (e)

Общую мощность также можно рассчитать несколькими способами. Выбирая P = IV и вводя полный ток, получаем

P = IV = (14,92 A) (12,0 В) = 179 Вт.

Обсуждение для (e)

Суммарная мощность, рассеиваемая резисторами, также 179 Вт:

P 1 + P 2 + P 3 = 144 Вт + 24,0 Вт + 11,1 Вт = 179 Вт

Это соответствует закону сохранения энергии.

Общее обсуждение

Обратите внимание, что как токи, так и мощность при параллельном подключении больше, чем для тех же устройств, подключенных последовательно.

Основные характеристики резисторов, подключенных параллельно
  1. Параллельное сопротивление определяется из [latex] \ frac {1} {{R} _ {\ text {p}}} = \ frac {1} {{R} _ {1}} + \ frac {1} { {R} _ {2}} + \ frac {1} {{R} _ {3}} + \ text {…} \\ [/ latex], и оно меньше любого отдельного сопротивления в комбинации.
  2. На каждый параллельно включенный резистор подается такое же полное напряжение источника, как и на него. (В системах распределения электроэнергии чаще всего используются параллельные соединения для питания бесчисленных устройств, обслуживаемых одним и тем же напряжением, и для того, чтобы они могли работать независимо.)
  3. Параллельные резисторы не получают суммарный ток каждый; они делят это.

Сочетания последовательного и параллельного

Более сложные соединения резисторов иногда представляют собой просто комбинации последовательного и параллельного. Они часто встречаются, особенно если учитывать сопротивление провода. В этом случае сопротивление провода включено последовательно с другими сопротивлениями, включенными параллельно. Комбинации последовательного и параллельного подключения можно свести к одному эквивалентному сопротивлению, используя технику, показанную на рисунке 4.Различные части идентифицируются как последовательные или параллельные, уменьшаются до их эквивалентов и далее уменьшаются до тех пор, пока не останется единственное сопротивление. Процесс более трудоемкий, чем трудный.

Рис. 4. Эта комбинация из семи резисторов имеет как последовательные, так и параллельные части. Каждый из них идентифицируется и приводится к эквивалентному сопротивлению, а затем уменьшается до тех пор, пока не будет достигнуто одно эквивалентное сопротивление.

Самая простая комбинация последовательного и параллельного сопротивления, показанная на рисунке 4, также является наиболее поучительной, поскольку она используется во многих приложениях.Например, R 1 может быть сопротивлением проводов от автомобильного аккумулятора к его электрическим устройствам, которые включены параллельно. R 2 и R 3 могли быть стартером и светом салона. Ранее мы предполагали, что сопротивление провода незначительно, но, когда это не так, оно имеет важные последствия, как показывает следующий пример.

Пример 3. Расчет сопротивления,

IR Падение, ток и рассеиваемая мощность: объединение последовательных и параллельных цепей

На рис. 5 показаны резисторы из двух предыдущих примеров, подключенные другим способом — комбинацией последовательного и параллельного подключения.Мы можем считать R 1 сопротивлением проводов, ведущих к R 2 и R 3 . (а) Найдите полное сопротивление. (b) Что такое IR для R 1 ? (c) Найдите текущие I 2 через R 2 . (d) Какую мощность рассеивает R 2 ?

Рис. 5. Эти три резистора подключены к источнику напряжения, так что R 2 и R 3 параллельны друг другу, и эта комбинация последовательно с R 1 .

Стратегия и решение для (а)

Чтобы найти полное сопротивление, отметим, что R 2 и R 3 находятся параллельно, а их комбинация R p последовательно с R 1 . Таким образом, полное (эквивалентное) сопротивление этой комбинации составляет

.

R до = R 1 + R p .

Сначала мы находим R p , используя уравнение для параллельных резисторов и вводя известные значения:

[латекс] \ frac {1} {{R} _ {\ text {p}}} = \ frac {1} {{R} _ {2}} + \ frac {1} {{R} _ {3 }} = \ frac {1} {6 \ text {.} \ text {00} \ text {} \ Omega} + \ frac {1} {\ text {13} \ text {.} 0 \ text {} \ Omega} = \ frac {0.2436} {\ text {} \ Омега} \\ [/ латекс].

Инвертирование дает

[латекс] {R} _ {\ text {p}} = \ frac {1} {0,2436} \ text {} \ Omega = 4.11 \ text {} \ Omega \\ [/ latex].

Таким образом, общее сопротивление равно

.

R до = R 1 + R p = 1,00 Ом + 4,11 Ом = 5,11 Ом.

Обсуждение для (а)

Общее сопротивление этой комбинации является промежуточным между значениями чистой серии и чистой параллели (20.0 Ом и 0,804 Ом соответственно), найденные для тех же резисторов в двух предыдущих примерах.

Стратегия и решение для (b)

Чтобы найти падение IR в R 1 , отметим, что полный ток I протекает через R 1 . Таким образом, падение IR составляет

.

В 1 = ИК 1

Мы должны найти I , прежде чем сможем вычислить V 1 .Полный ток I находится с помощью закона Ома для схемы. То есть

[латекс] I = \ frac {V} {{R} _ {\ text {tot}}} = \ frac {\ text {12.0} \ text {V}} {5.11 \ text {} \ Omega} = 2.35 \ text {A} \\ [/ latex].

Вводя это в выражение выше, мы получаем

В 1 = IR 1 = (2,35 A) (1,00 Ом) = 2,35 В.

Обсуждение для (б)

Напряжение, приложенное к R 2 и R 3 , меньше полного напряжения на величину В 1 .Когда сопротивление провода велико, это может существенно повлиять на работу устройств, представленных R 2 и R 3 .

Стратегия и решение для (c)

Чтобы найти ток через R 2 , мы должны сначала найти приложенное к нему напряжение. Мы называем это напряжение В p , потому что оно приложено к параллельной комбинации резисторов. Напряжение, приложенное как к R 2 , так и к R 3 , уменьшается на величину В 1 , и поэтому оно составляет

.

В p = V V 1 = 12.0 В — 2,35 В = 9,65 В.

Теперь ток I 2 через сопротивление R 2 находится по закону Ома:

[латекс] {I} _ {2} = \ frac {{V} _ {\ text {p}}} {{R} _ {2}} = \ frac {9.65 \ text {V}} {6.00 \ текст {} \ Omega} = 1,61 \ text {A} \\ [/ latex].

Обсуждение для (c)

Ток меньше, чем 2,00 А, которые протекали через R 2 , когда он был подключен параллельно к батарее в предыдущем примере параллельной цепи.

Стратегия и решение для (d)

Мощность, рассеиваемая R 2 , определяется

P 2 = ( I 2 ) 2 R 2 = (1,61 A) 2 (6,00 Ом) = 15,5 Вт

Обсуждение для (d)

Мощность меньше 24,0 Вт, рассеиваемых этим резистором при параллельном подключении к источнику 12,0 В.

Одним из следствий этого последнего примера является то, что сопротивление в проводах снижает ток и мощность, подаваемую на резистор.Если сопротивление провода относительно велико, как в изношенном (или очень длинном) удлинителе, то эти потери могут быть значительными. Если потребляется большой ток, падение IR в проводах также может быть значительным.

Например, когда вы роетесь в холодильнике и включается мотор, свет холодильника на мгновение гаснет. Точно так же вы можете увидеть тусклый свет в салоне, когда вы запускаете двигатель вашего автомобиля (хотя это может быть связано с сопротивлением внутри самой батареи).

То, что происходит в этих сильноточных ситуациях, показано на рисунке 6. Устройство, обозначенное номером R 3 , имеет очень низкое сопротивление, поэтому при его включении протекает большой ток. Этот увеличенный ток вызывает большее падение IR в проводах, представленных R 1 , уменьшая напряжение на лампочке (которое составляет R 2 ), которое затем заметно гаснет.

Рис. 6. Почему гаснет свет при включении большого прибора? Ответ заключается в том, что большой ток, потребляемый двигателем прибора, вызывает значительное падение напряжения в проводах и снижает напряжение на свету.

Проверьте свое понимание

Можно ли любую произвольную комбинацию резисторов разбить на последовательную и параллельную? Посмотрите, сможете ли вы нарисовать принципиальную схему резисторов, которые нельзя разбить на комбинации последовательно и параллельно.

Решение Нет, есть много способов подключения резисторов, которые не являются комбинациями последовательного и параллельного, включая петли и переходы. В таких случаях правила Кирхгофа, которые будут включены в Правила Кирхгофа, позволят вам проанализировать схему.

Стратегии решения проблем для последовательных и параллельных резисторов
  1. Нарисуйте четкую принципиальную схему, обозначив все резисторы и источники напряжения. Этот шаг включает список известных проблем, поскольку они отмечены на вашей принципиальной схеме.
  2. Точно определите, что необходимо определить в проблеме (определите неизвестные). Письменный список полезен.
  3. Определите, включены ли резисторы последовательно, параллельно или в комбинации последовательно и параллельно.Изучите принципиальную схему, чтобы сделать эту оценку. Резисторы включены последовательно, если через них должен последовательно проходить один и тот же ток.
  4. Используйте соответствующий список основных функций для последовательных или параллельных соединений, чтобы найти неизвестные. Есть один список для серий, а другой — для параллелей. Если ваша проблема представляет собой комбинацию последовательного и параллельного соединения, уменьшайте ее поэтапно, рассматривая отдельные группы последовательных или параллельных соединений, как это сделано в этом модуле и примерах. Особое примечание: при обнаружении R необходимо проявлять осторожность.
  5. Проверьте, являются ли ответы разумными и последовательными. Единицы и числовые результаты должны быть разумными. Общее последовательное сопротивление должно быть больше, а общее параллельное сопротивление, например, должно быть меньше. Мощность должна быть больше для одних и тех же устройств, подключенных параллельно, по сравнению с последовательными и т. Д.

Сводка раздела

Концептуальные вопросы

1. Переключатель имеет переменное сопротивление, которое почти равно нулю в замкнутом состоянии и очень велико в разомкнутом состоянии, и он включен последовательно с устройством, которым он управляет.Объясните влияние переключателя на рис. 7 на ток в разомкнутом и замкнутом состоянии.

Рис. 7. Переключатель обычно включается последовательно с источником сопротивления и напряжения. В идеале переключатель имеет почти нулевое сопротивление в замкнутом состоянии, но имеет чрезвычайно большое сопротивление в разомкнутом состоянии. (Обратите внимание, что на этой диаграмме скрипт E представляет напряжение (или электродвижущую силу) батареи.)

2. Какое напряжение на разомкнутом переключателе на Рисунке 7?

3. На разомкнутом переключателе есть напряжение, как на Рисунке 7.Почему же тогда мощность, рассеиваемая разомкнутым переключателем, мала?

4. Почему мощность, рассеиваемая замкнутым переключателем, как на Рисунке 7, мала?

5. Студент в физической лаборатории по ошибке подключил электрическую лампочку, батарею и выключатель, как показано на рисунке 8. Объясните, почему лампочка горит, когда выключатель разомкнут, и гаснет, когда выключатель замкнут. (Не пытайтесь — батарея сильно разряжается!)

Рис. 8. Ошибка подключения. Включите этот переключатель параллельно устройству, обозначенному [латекс] R [/ латекс].(Обратите внимание, что на этой диаграмме скрипт E представляет напряжение (или электродвижущую силу) батареи.)

6. Зная, что сила удара зависит от величины тока, протекающего через ваше тело, вы бы предпочли, чтобы он был включен последовательно или параллельно с сопротивлением, таким как нагревательный элемент тостера, если он потрясен им? Объяснять.

7. Были бы ваши фары тусклыми при запуске двигателя автомобиля, если бы провода в вашем автомобиле были сверхпроводниками? (Не пренебрегайте внутренним сопротивлением батареи.) Объяснять.

8. Некоторые гирлянды праздничных огней соединены последовательно для экономии затрат на проводку. В старой версии использовались лампочки, которые при перегорании прерывали электрическое соединение, как открытый выключатель. Если одна такая лампочка перегорит, что случится с остальными? Если такая цепочка работает от 120 В и имеет 40 одинаковых лампочек, каково нормальное рабочее напряжение каждой? В более новых версиях используются лампы, которые при перегорании замыкаются накоротко, как замкнутый выключатель. Если одна такая лампочка перегорит, что случится с остальными? Если такая цепочка работает от 120 В и в ней осталось 39 идентичных лампочек, каково тогда рабочее напряжение каждой?

9.Если две бытовые лампочки мощностью 60 и 100 Вт подключить последовательно к бытовой электросети, какая из них будет ярче? Объяснять.

10. Предположим, вы проводите физическую лабораторию, которая просит вас вставить резистор в цепь, но все прилагаемые резисторы имеют большее сопротивление, чем запрошенное значение. Как бы вы соединили доступные сопротивления, чтобы попытаться получить меньшее запрошенное значение?

11. Перед Второй мировой войной некоторые радиостанции получали питание через «шнур сопротивления», который имел значительное сопротивление.Такой резистивный шнур снижает напряжение до желаемого уровня для ламп радиоприемника и т.п., и это экономит расходы на трансформатор. Объясните, почему шнуры сопротивления нагреваются и тратят энергию при включенном радио.

12. У некоторых лампочек есть три уровня мощности (не включая ноль), полученные от нескольких нитей накала, которые индивидуально переключаются и соединяются параллельно. Какое минимальное количество нитей нити необходимо для трех режимов мощности?

Задачи и упражнения

Примечание. Можно считать, что данные, взятые из цифр, имеют точность до трех значащих цифр.

1. (a) Каково сопротивление десяти последовательно соединенных резисторов сопротивлением 275 Ом? (б) Параллельно?

2. (a) Каково сопротивление последовательно соединенных резисторов 1,00 × 10 2 Ом, 2,50 кОм и 4,00 кОм? (б) Параллельно?

3. Какое наибольшее и наименьшее сопротивление можно получить, соединив резисторы на 36,0 Ом, 50,0 Ом и 700 Ом?

4. Тостер на 1800 Вт, электрическая сковорода на 1400 Вт и лампа на 75 Вт подключены к одной розетке в цепи 15 А, 120 В.(Три устройства работают параллельно, если они подключены к одной розетке.) а) Какой ток потребляет каждое устройство? (b) Перегорит ли эта комбинация предохранитель на 15 А?

5. Фара мощностью 30,0 Вт и стартер мощностью 2,40 кВт обычно подключаются параллельно в систему на 12,0 В. Какую мощность потребляли бы одна фара и стартер при последовательном подключении к батарее 12,0 В? (Не обращайте внимания на любое другое сопротивление в цепи и любое изменение сопротивления в двух устройствах.)

6.(a) Учитывая батарею 48,0 В и резисторы 24,0 Ом и 96,0 Ом, найдите для каждого из них ток и мощность при последовательном соединении. (b) Повторите, когда сопротивления включены параллельно.

7. Ссылаясь на пример комбинирования последовательных и параллельных цепей и рисунок 5, вычислите I 3 двумя следующими способами: (a) по известным значениям I и I 2 ; (б) используя закон Ома для R 3 . В обеих частях явно показано, как вы следуете шагам, описанным в Стратегии решения проблем для последовательных и параллельных резисторов выше.

Рис. 5. Эти три резистора подключены к источнику напряжения, так что R 2 и R 3 параллельны друг другу, и эта комбинация последовательно с R 1 .

8. Ссылаясь на рисунок 5: (a) Вычислите P 3 и обратите внимание на его сравнение с P 3 , найденным в первых двух примерах задач в этом модуле. (b) Найдите полную мощность, отдаваемую источником, и сравните ее с суммой мощностей, рассеиваемых резисторами.

9. См. Рисунок 6 и обсуждение затемнения света при включении тяжелого прибора. (a) Учитывая, что источник напряжения составляет 120 В, сопротивление провода составляет 0,400 Ом, а номинальная мощность лампы составляет 75,0 Вт, какая мощность будет рассеиваться лампой, если при включении двигателя через провода пройдет в общей сложности 15,0 А? Предположите незначительное изменение сопротивления лампы. б) Какая мощность потребляет двигатель?

Рис. 6. Почему гаснет свет при включении большого прибора? Ответ заключается в том, что большой ток, потребляемый двигателем прибора, вызывает значительное падение напряжения в проводах и снижает напряжение на свету.

10. Линия электропередачи на 240 кВ, имеющая 5,00 × 10 2 , подвешена к заземленным металлическим опорам с помощью керамических изоляторов, каждый из которых имеет сопротивление 1,00 × 10 9 Ом (рис. 9 (а)). Какое сопротивление на землю у 100 изоляторов? (b) Рассчитайте мощность, рассеиваемую 100 из них. (c) Какая доля мощности, переносимой линией, составляет это? Ясно покажите, как вы следуете шагам, описанным выше в стратегии решения проблем для последовательных и параллельных резисторов .

Рис. 9. Высоковольтная (240 кВ) линия электропередачи 5,00 × 10 2 подвешена к заземленной металлической опоре электропередачи. Ряд керамических изоляторов обеспечивает сопротивление 1,00 × 10 9 Ом каждый.

11. Покажите, что если два резистора R 1 и R 2 объединены, и один из них намного больше другого ( R 1 >> R 2 ): (a ) Их последовательное сопротивление почти равно большему сопротивлению R 1 .(б) Их параллельное сопротивление почти равно меньшему сопротивлению R 2 .

12. Необоснованные результаты Два резистора, один с сопротивлением 145 Ом, подключены параллельно, чтобы получить общее сопротивление 150 Ом. а) Каково значение второго сопротивления? б) Что неразумного в этом результате? (c) Какие предположения необоснованны или непоследовательны?

13. Необоснованные результаты Два резистора, один из которых имеет сопротивление 900 кОм, соединены последовательно, чтобы получить общее сопротивление 0.500 МОм. а) Каково значение второго сопротивления? б) Что неразумного в этом результате? (c) Какие предположения необоснованны или непоследовательны?

Глоссарий

серия:
последовательность резисторов или других компонентов, включенных в цепь один за другим
резистор:
компонент, обеспечивающий сопротивление току, протекающему через электрическую цепь
сопротивление:
, вызвав потерю электроэнергии в цепи
Закон Ома:
соотношение между током, напряжением и сопротивлением в электрической цепи: В = IR
напряжение:
электрическая потенциальная энергия на единицу заряда; электрическое давление, создаваемое источником питания, например аккумулятором
падение напряжения:
потеря электроэнергии при прохождении тока через резистор, провод или другой компонент
текущий:
поток заряда через электрическую цепь мимо заданной точки измерения
Закон Джоуля:
соотношение между потенциальной электрической мощностью, напряжением и сопротивлением в электрической цепи, определяемое следующим образом: [latex] {P} _ {e} = \ text {IV} [/ latex]
параллельно:
подключение резисторов или других компонентов в электрической цепи таким образом, что каждый компонент получает одинаковое напряжение от источника питания; часто изображается на диаграмме в виде лестницы, где каждый компонент находится на ступеньке лестницы

Избранные решения проблем и упражнения

1.(а) 2,75 кОм (б) 27,5 Ом

3. (а) 786 Ом (б) 20,3 Ом

5. 29,6 Вт

7. (а) 0,74 А (б) 0,742 А

9. (а) 60,8 Вт (б) 3,18 кВт

11. (a) [латекс] \ begin {array} {} {R} _ {\ text {s}} = {R} _ {1} + {R} _ {2} \\ \ Rightarrow {R} _ {\ text {s}} \ приблизительно {R} _ {1} \ left ({R} _ {1} \ text {>>} {R} _ {2} \ right) \ end {array} \\ [/ латекс]

(b) [латекс] \ frac {1} {{R} _ {p}} = \ frac {1} {{R} _ {1}} + \ frac {1} {{R} _ {2} } = \ frac {{R} _ {1} + {R} _ {2}} {{R} _ {1} {R} _ {2}} \\ [/ latex],

, так что

[латекс] \ begin {array} {} {R} _ {p} = \ frac {{R} _ {1} {R} _ {2}} {{R} _ {1} + {R} _ {2}} \ приблизительно \ frac {{R} _ {1} {R} _ {2}} {{R} _ {1}} = {R} _ {2} \ left ({R} _ {1 } \ text {>>} {R} _ {2} \ right) \ text {.} \ end {array} \\ [/ latex]

13. (a) –400 кОм (b) Сопротивление не может быть отрицательным. (c) Считается, что последовательное сопротивление меньше, чем у одного из резисторов, но должно быть больше, чем у любого из резисторов.

электричество — Почему падение напряжения на каждом резисторе в параллельной цепи остается неизменным?

Тот факт, что напряжение — это энергия, приходящаяся на заряд, не обязательно означает, что это кинетическая энергия, а скорее потенциальная энергия. Если электроны не будут испытывать трения (в отличие от вашего заявления о том, что они «теряют больше кинетической энергии, сталкиваясь с другими атомами»), различие между потенциальной и кинетической энергией становится несущественным, потому что один просто преобразуется в другой — электроны просто ускоряются. в электрическом поле.Это примерно так, например, для электронного пучка в электронно-лучевой трубке. Следовательно, кинетическая энергия электронов на конце электронно-лучевой трубки обычно выражается в «электрон-вольтах».

Однако, если электроны испытывают трение (также известное как сопротивление, «теряют больше кинетической энергии, сталкиваясь с другими атомами»), их скоординированное движение непрерывно преобразуется в нескоординированное движение, то есть статистическое тепловое движение. Это означает, что вместо непрерывного преобразования потенциальной энергии в макроскопическую кинетическую энергию (что эквивалентно утверждению о том, что электроны ускоряются все больше и больше), они в конечном итоге перемещаются по с постоянной скоростью (при этом любая энергия, полученная от потенциала, почти немедленно преобразуется в тепло вместо кинетической энергии).Постоянная скорость также может быть объединена с плотностью электронов, чтобы выразить плотность тока. Это «сопротивление» электронов нельзя вывести только на основании напряжения (также известного как потенциальная энергия) и законов механики Ньютона, но оно является следствием закона Ома (который описывает связь между напряжением и равновесной скоростью / током электронов). или, другими словами, диссипация энергии / трение / тепловой беспорядок, когда «электроны натыкаются на атомы»). Закон Ома чем-то сравним с вязкостью в механике жидкости, которая также нарушает сохранение (механической) энергии.

Короче говоря: вместо того, чтобы терять «больше кинетической энергии» в резисторе с большим номиналом, электроны рассеивают ту же энергию (потенциальную), но в резисторе с большим номиналом (где больше столкновений с атомами ) их равновесная скорость / ток соответственно меньше, так что они теряют меньшее количество кинетической энергии при каждом ударе (см. https://en.wikipedia.org/wiki/Mean_free_path для более подробного объяснения).

В качестве примерной аналогии подумайте о вас в городской пробке.Напряжение соответствует вашему времени (например, чтобы попасть на прием). Если вы выберете путь через центр города, ваша способность к захвату и ускорению будет больше ограничена повышенной плотностью движения там. Следовательно, ваша средняя скорость около центра города будет ниже, потому что вам придется чаще останавливаться за другой машиной. Количество кинетической энергии, которую вы теряете, будет меньше во время каждой отдельной остановки. Если вы выберете путь через окраины, у вас может быть больше возможностей быстрее добраться до цели, поэтому ваша средняя скорость выше.Но если перед вами остановится машина, вы потеряете больше кинетической энергии. В любом случае давление (то есть ваше желание вовремя добраться до цели) остается прежним. Это не идеальная аналогия, но я надеюсь, что она объясняет ключевые моменты.

Описание цепей постоянного тока серии

и параллельных (примеры включены)

Что такое электрическая цепь?

Электрическая цепь представляет собой комбинацию двух или более электрических компонентов, которые соединены между собой токопроводящими дорожками.Электрические компоненты могут быть активными компонентами или неактивными компонентами или их комбинацией.

Что такое цепь постоянного тока?

Электроэнергия бывает двух видов — постоянного (DC) и переменного (AC) тока. Цепь, которая имеет дело с постоянным током или постоянным током, называется цепью постоянного тока , , а цепь, которая имеет дело с переменным током или переменным током, называется цепью переменного тока.

Компоненты электрической цепи постоянного тока в основном резистивные, тогда как компоненты цепи переменного тока могут быть как реактивными, так и резистивными.

Любую электрическую цепь можно разделить на три разные группы — последовательную, параллельную и последовательно-параллельную. Так, например, в случае постоянного тока цепи также можно разделить на три группы, такие как последовательная цепь постоянного тока , параллельная цепь постоянного тока, и последовательная и параллельная .

Что такое последовательная цепь постоянного тока?

Когда все резистивные компоненты цепи постоянного тока соединены встык, образуя единый путь для протекающего тока, тогда цепь называется последовательностью постоянного тока .Способ соединения компонентов встык известен как последовательное соединение.

Предположим, что у нас есть n резисторов R 1 , R 2 , R 3 ………… R n , и они соединены между собой, то есть последовательно. Если эта последовательная комбинация подключена к источнику напряжения, ток начинает течь по этому единственному пути.

Поскольку резисторы соединены встык, ток сначала входит в R 1 , затем этот же ток входит в R 2 , затем R 3 и, наконец, достигает R n от ток которого поступает на отрицательные выводы источника напряжения.

Таким образом, одинаковый ток проходит через все последовательно включенные резисторы. Следовательно, можно сделать вывод, что в цепи постоянного тока серии , одинаковый ток течет через все части электрической цепи.

Опять же, согласно закону Ома, падение напряжения на резисторе является произведением его электрического сопротивления и тока, протекающего через него.

Здесь ток через каждый резистор одинаков, следовательно, падение напряжения на каждом резисторе пропорционально значению его электрического сопротивления.

Если сопротивления резисторов не равны, падение напряжения на них также не будет равным. Таким образом, каждый резистор имеет свое индивидуальное падение напряжения в цепи постоянного тока серии .

Электрическая последовательная цепь постоянного тока с тремя резисторами

Ниже приведен рисунок последовательной цепи постоянного тока с тремя резисторами. Течение тока показано здесь движущейся точкой. Обратите внимание, что это всего лишь концептуальное представление.

Пример цепи постоянного тока серии

Предположим, что три резистора R 1 , R 2, и R 3 подключены последовательно к источнику напряжения V (измеряется в вольтах), как показано на рисунке.Пусть ток I (количественно выраженный как Ампера) протекает через последовательную цепь. Теперь по закону Ома
Падение напряжения на резисторе R 1 , В 1 = IR 1
Падение напряжения на резисторе R 2 , В 2 = IR 2
Падение напряжения на резисторе R 3 , В 3 = IR 3
Падение напряжения во всей последовательной цепи постоянного тока,
В = падение напряжения на резисторе R 1 + падение напряжения на резисторе R 2 + падение напряжения через резистор R 3



Согласно закону Ома электрическое сопротивление электрической цепи определяется как V ⁄ I, то есть R.Следовательно,

Итак, эффективное сопротивление последовательной цепи постоянного тока составляет. Из приведенного выше выражения можно сделать вывод, что, когда несколько резисторов соединены последовательно, эквивалентное сопротивление последовательной комбинации является арифметической суммой их индивидуальных сопротивлений.
Из приведенного выше обсуждения вытекают следующие моменты:

  1. Когда несколько электрических компонентов соединены последовательно, один и тот же ток течет через все компоненты цепи.
  2. Приложенное напряжение в последовательной цепи равно сумме падений напряжения на каждом компоненте.
  3. Падение напряжения на отдельных компонентах прямо пропорционально значению их сопротивления.

Что такое параллельная цепь постоянного тока?

Когда два или более электрических компонента соединены таким образом, что один конец каждого компонента подключен к общей точке, а другой конец подключен к другой общей точке, электрические компоненты считаются подключенными параллельно, и такие электрическая цепь постоянного тока упоминается как параллельная цепь постоянного тока .

В этой схеме у каждого компонента будет одинаковое падение напряжения на них, и оно будет точно равно напряжению, которое возникает между двумя общими точками, где компоненты соединены.

Также в параллельной цепи постоянного тока , ток имеет несколько параллельных путей через эти параллельно соединенные компоненты, поэтому ток схемы будет разделен на столько путей, сколько количество компонентов.

Здесь, в этой электрической цепи, падение напряжения на каждом компоненте одинаковое.Опять же, согласно закону Ома, падение напряжения на любом резистивном компоненте равно произведению его электрического сопротивления и тока через него.

Поскольку падение напряжения на всех компонентах, подключенных параллельно, одинаково, ток через них обратно пропорционален значению их сопротивления.

Электрическая параллельная цепь постоянного тока с тремя резисторами

Ниже приведен рисунок параллельной цепи постоянного тока с тремя резисторами. Течение тока показано здесь движущейся точкой.Обратите внимание, что это всего лишь концептуальное представление.

Примеры параллельной цепи постоянного тока

Предположим, что три резистора R 1 , R 2, и R 3 подключены параллельно к источнику напряжения V (вольт), как показано на рисунке. Пусть I (Ампер) будет общим током цепи, который разделен на ток I 1 , I 2, и I 3 , протекающий через R 1 , R 2, и R 3 соответственно.Теперь по закону Ома:
Падение напряжения на резисторе R 1 , V = I 1 .R 1
Падение напряжения на резисторе R 2 , V = I 2 .R 2
Падение напряжения на резисторе R 3 , V = I 3 .R 3
Падение напряжения во всей параллельной цепи постоянного тока,
В = падение напряжения на резисторе R 1 = падение напряжения на резисторе R 2 = падение напряжения на резисторе R 3
⇒ V = I 1 .R 1 = I 2 .R 2 = I 3 .R 3



Таким образом, когда несколько резисторов подключены параллельно, эквивалентное сопротивление равно арифметическая сумма обратных величин их индивидуальных сопротивлений.
Из приведенного выше обсуждения параллельной цепи постоянного тока мы можем прийти к следующему выводу:

  1. Падения напряжения одинаковы на всех компонентах, соединенных параллельно.
  2. Ток через отдельные компоненты, подключенные параллельно, обратно пропорционален их сопротивлению.
  3. Полный ток цепи — это арифметическая сумма токов, проходящих через отдельные компоненты, соединенные параллельно.
  4. Обратное эквивалентное сопротивление равно сумме обратных сопротивлений отдельных компонентов, соединенных параллельно.

Объединение последовательных и параллельных цепей

До сих пор мы обсуждали последовательные цепи постоянного тока и параллельные цепи постоянного тока отдельно, но на практике электрическая цепь обычно представляет собой комбинацию как последовательных цепей, так и параллельных цепей.

Добавить комментарий

Ваш адрес email не будет опубликован. Обязательные поля помечены *